最新LSAT考试逻辑推理模拟试题(三)
最新LSAT逻辑推理练习题
最新LSAT逻辑推理练习题我们都知道一般法学院要求151分,知名的法学院往往要求163分以上。
若申请者的LSAT成绩没有达到一定要求,有些学校甚至不会看其申请材料。
怎么办?我们只能练题来提高自己的分数了,LSAT逻辑推理练习题分享给大家!注意:LSAT考试满分为180分,最低分为120分,其计算方法是根据选择的正确的答案的数目来确定。
总共99-101道,题选对25个左右,分数大概为130分;选对39个左右,分数大概为140分;选对55个,分数大概为150分;选对72个,分数为160;选对87个,分数为170分;选对98个以上,分数为满分180分。
LSAT 逻辑游戏题On Wednesday, a legislator remembers that she must vote on seven bills-defense, environment, free trade, gun control, health care, immigration, and judicial activism-by the end of the week. Because the legislator wants to align herself with a major political party, she will vote on the seven bills in accordance with the following conditions:She votes for the gun control bill only if she votes against the environment bill.Unless she votes against the judicial activism bill, she will vote for the immigration bill.She will vote for either the environment bill, the judicial activism bill, or both.She votes for the gun control bill if she votes for both the health care bill and the defense bill.1. Which one of the following could be a complete and accurate list of the bills the legislator votes against?(A) free trade, gun control, immigration, judicial activism(B) defense, free trade, gun control, health care, immigration(C) free trade, health care, immigration, judicial activism(D) environment, gun control, health care, judicial activism(E) defense, environment, gun control, health care2. If the legislator votes against the judicial activism bill, then which one of the following CANNOT be true?(A) She votes against both the defense bill and the gun control bill.(B) She votes against both the gun control bill and the health care bill.(C) She votes for both the health care bill and the defense bill.(D) She votes for both the health care bill and the environment bill.(E) She votes for both the environment bill and the free trade bill.3. Which one of the following CANNOT be true?(A) The legislator votes for neither the gun control bill nor the immigration bill.(B) The legislator votes for neither the environment bill nor the immigration bill.(C) The legislator votes for neither the gun control bill nor the health care bill.(D) The legislator votes for neither the health care bill nor the free trade bill.(E) The legislator votes for neither the free trade bill nor the immigration bill.4. If the legislator votes against the immigration bill, then which one of the following is the minimum number of the seven bills she must also vote against?(A) one(B) two(C) three(D) four(E) five5. If the legislator votes for the gun control bill, then which one of the following must be true?(A) She votes for the health care bill or the defense bill.(B) She votes against the health care bill or the defense bill.(C) She votes against the judicial activism bill.(D) She votes for the immigration bill.(E) She votes against the immigration bill.6. If the legislator votes against the judicial activism bill, then each of the following could be true EXCEPT:(A) She votes against the health care bill and the defense bill.(B) She votes for the health care bill and the defense bill.(C) She votes against the health care bill and the gun control bill.(D) She votes against the free trade bill and the gun control bill.(E) She votes for the environment bill and the free trade bill.7. Suppose the condition is added that if the legislator votes for the free trade bill, then she will vote against the judicial activism bill. If all other conditions remain in effect, then each of the following must be true EXCEPT:(A) If she votes for the free trade bill, then she votes against the gun control bill.(B) If she votes against the environment bill, then she also votes against the free trade bill.(C) If she votes against the immigration bill, then she votes for the free trade bill.(D) If she votes against the judicial activism bill, then she also votes against the gun control bill.(E) If she votes against the immigration bill, then she also votes against at least three other bills.除了刷真题以外,我觉得大家忽略了一点:这是一个临场发挥的考试,因此考试的状态是最关键的。
最新LSAT考试逻辑推理模拟试题(三)
最新 LSAT 考试逻辑推理模拟试题(三)TEST 2 SECTION II Time 35 minutes 24 Questions Directions: The questions in this section are based on the reasoning contained in brief statements or passages.1.Some people believe that witnessing violence in movies will discharge aggressive energy.Does watching someone else eat fill one’s own stomach? In which one of the following does the reasoning most closely parallel that employed in the passage? (A) Some people think appropriating supplies at work for their own personal use is morally wrong. Isn’t shoplifting morally wrong? (B) Some people think nationalism is defensible. Hasn’t nationalism been the excuse for committing abominable crimes? (C) Some people think that boxing is fixed just because wrestling usually is. Are the two sports managed by the same sort of people? (D) Some people think that economists can control inflation. Can meteorologists make the sun shine?(D) (E) Some people think workaholics are compensating for a lack of interpersonal skills. However, aren’t most doctors workaholics?2.Ann: All the campers at Camp Winnehatchee go to Tri-Cities High SchoolBill: That’s not true. Some Tri-Cities students are campers at Camp Lakemont. Bill’s answer can be best explained on the assumption that he has interpreted Ann’s remark to mean that (A) most of the campers at Camp Lakemont come from high schools other than Tri-Cities (B) most Tri-Cities High School students are campers at Camp Winnehatchee (C) some Tri-Cities High School students have withdrawn from Camp Lakemont (D) all Tri-Cities High School students have withdrawn from Camp Lakemont(E) (E) only campers at Camp Winnehatchee are students at Tri-Cities High School3.More than a year ago, the city announced that police would crack down on illegally parkedcars and that resources would be diverted from writing speeding tickets to ticketing illegally parked cars. But no crackdown has taken place. The police chief claims that resources have had tobe diverted from writing speeding tickets to combating the city’s staggering drug problem. Yet the police are still writing as many speeding tickets as ever. Therefore, the excuse about resources being tied up in fighting drug-related crime simply is not true. The conclusion in the passage depends on the assumption that (A) every member of the police force is qualified to work on combating the city’s drug problem (B) drug-related crime is not as serious a problem for the city as the police chief claims it is (C) writing speeding tickets should be as important a priority for the city as combating drug-related crime (D) the police could be cracking down on illegally parked cars and combating the drug problem without having to reduce writing speeding tickets(E) (E) the police cannot continue writing as many speeding tickets as ever while diverting resources to combating drug-related crime4.Dried grass clippings mixed into garden soil gradually decompose, providing nutrients forbeneficial soil bacteria. This results in better-than-average plant growth. Yet mixing fresh grass clippings into garden soil usually causes poorer-than-average plant growth. Which one of the following, if true, most helps to explain the difference in plant growth described above? (A) The number of beneficial soil bacteria increases whenever any kind of plant material is mixed into garden soil. (B) Nutrients released by dried grass clippings are immediately available to beneficial soil bacteria. (C) Some dried grass clippings retain nutrients originally derived from commercial lawn fertilizers, and thus provide additional enrichment to the soil. (D) Fresh grass clippings mixed into soil decompose rapidly, generating high levels of heat that kill beneficial soil bacteria.(D) (E) When a mix of fresh and dried grass clippings is mixed into garden soil, plant growth often decreases.5.A gas tax of one cent per gallon would raise one billion dollars per year at currentconsumption rates. Since a tax of fifty cents per gallon would therefore raise fifty billion dollars per year, it seems a perfect way to deal with the federal budget deficit. This tax would have the additional advantage that the resulting drop in the demand for gasoline would be ecologically sound and would keep our country from being too dependent on foreign oil producers. Which one of the following most clearly identifies an error in the author’s reasoning? (A) The author cites irrelevant data. (B) The author relies on incorrect current consumption figures. (C) The author makes incompatible assumptions.(D) The author mistakes an effect for a cause.(C) (E) The author appeals to conscience rather than reason.6.As symbols of the freedom of the wilderness, bald eagles have the unique capacity to inspirepeople and foster in them a sympathetic attitude toward the needs of other threatened species. Clearly, without that sympathy and the political will it engenders, the needs of more obscure species will go unmet. The conservation needs of many obscure species can only be met by beginning with the conservation of this symbolic species, the bald eagle. Which one of the following is the main point of the passage as a whole? (A) Because bald eagles symbolize freedom, conservation efforts should be concentrated on them rather than on other, more obscure species. (B) The conservation of bald eagles is the first necessary step in conserving other endangered species. (C) Without increased public sympathy for conservation, the needs of many symbolic species will go unmet. (D) People’s love of the wilderness can be used to engender political support for conservation efforts.(B) (E) Other threatened species do not inspire people or foster sympathy as much as do bald eagles. 来源:考试大-LSAT 7. There is no reason why the work of scientists has to be officially confirmed before beingpublished. There is a system in place for the confirmation or disconfirmation of scientific finding, namely, the replication of results by other scientists. Poor scientific work on the part of any one scientist, which can include anything from careless reporting practices to fraud, is not harmful. It will be exposed and rendered harmless when other scientists conduct the experiments and obtain disconfirmatory results. Which one of the following, if true, would weaken the argument? (A) Scientific experiments can go unchallenged for many years before they are replicated. (B) Most scientists work in universities, where their work is submitted to peer review before publication. (C) Most scientists are under pressure to make their work accessible to the scrutiny of replication. (D) In scientific experiments, careless reporting is more common than fraud.(A) (E) Most scientists work as part of a team rather than alone.8.Alice: Quotas on automobile imports to the United States should be eliminated. Thendomestic producers would have to compete directly with Japanese manufacturers and would be forced to produce higher-quality cars. Such competition would be good for consumers. David: You fail to realize, Alice, that quotas on automobile imports are pervasive worldwide.Since German, Britain, and France have quotas, so should the United States. Which one of the following most accurately characterizes David’s response to Alice’s statement? (A) David falsely accuses Alice of contradicting herself. (B) David unfairly directs his argument against Alice personally. (C) David uncovers a hidden assumption underlying Alice’s position. (D) David takes a position that is similar to the one Alice has taken.(E) (E) David fails to address the reasons Alice cites in favor of her conclusion.9.Governments have only one response to public criticism of socially necessary services:regulation of the activity of providing those services. But governments inevitably make the activity more expensive by regulating it, and that is particularly troublesome in these times of strained financial resources. However, since public criticism of child-care services has undermined all confidence in such services, and since such services are socially necessary, the government is certain to respond. Which one of the following statements can be inferred from the passage? (A) The quality of child care will improve. (B) The cost of providing child-care services will increase. (C) The government will use funding to foster advances in child care. (D) If public criticism of policy is strongly voiced, the government is certain to respond.(B) (E) If child-care services are not regulated, the cost of providing child care will not increase. Advertisers are often criticized for their unscrupulous manipulation of people’s tastes and10.wants. There is evidence, however, that some advertisers are motivated by moral as well as financial considerations. A particular publication decided to change its image from being a family newspaper to concentrating on sex and violence, thus appealing to a different readership. Some advertisers withdrew their advertisements from the publication, and this must have been because they morally disapproved of publishing salacious material. Which one of the following, if true, would most strengthen the argument? (A) The advertisers switched their advertisements to other family newspapers. (B) Some advertisers switched from family newspapers to advertise in the changed publication. (C) The advertisers expected their product sales to increase if they stayed with the changed publication, but to decrease if they withdrew. (D) People who generally read family newspapers are not likely to buy newspapers that concentrate on sex and violence.(C) (E) It was expected that the changed publication would appeal principally to those in a different income group.11. “If the forest continues to disappear at its present pace, the koala will approach extinction,” said the biologist. “So all that is needed to save the koala is to stop deforestation,” said the politician. Which one of the following statements is consistent with the biologist’s claim but not with the politician’s claim? (A) Deforestation continues and the koala becomes extinct. (B) Deforestation is stopped and the koala becomes extinct. (C) Reforestation begins and the koala survives. (D) Deforestation is slowed and the koala survives.(B) (E) Deforestation is slowed and the koala approaches extinction.12. People have long been fascinated by the paranormal. Over the years, numerous researchers have investigated telepathy only to find that conclusive evidence for its existence has persistently evaded them. Despite this, there are still those who believe that there must be “something in it” since some research seems to support the view that telepathy exist. However, it can often be shown that other explanations that do comply with known laws can be given. Therefore, it is premature to conclude that telepathy is an alternative means of communication. In the passage, the author (A) supports the conclusion by pointing to the inadequacy of evidence for the opposite view (B) supports the conclusion by describing particular experiments (C) supports the conclusion by overgeneralizing from a specific piece of evidence (D) draws a conclusion that is not supported by the premises(A) (E) rephrases the conclusion without offering any support for it 来源:考试大-LSAT 13. If retail stores experience a decrease in revenues during this holiday season, then either attitudes toward extravagant gift-giving have changed or prices have risen beyond the level most people can afford. If attitudes have changed, then we all have something to celebrate this season. If prices have risen beyond the level most people can afford, then it must be that salaries have not kept pace with rising prices during the past year. Assume the premises above to be true. If salaries have kept pace with rising prices during the past year, which one of the following must be true? (A) Attitudes toward extravagant gift-giving have changed. (B) Retail stores will not experience a decrease in retail sales during this holiday season. (C) Prices in retail stores have not risen beyond the level that most people can afford during this holiday season. (D) Attitudes toward extravagant gift-giving have not changed, and stores will not experience a decrease in revenues during this holiday season.(C)(E) Either attitudes toward extravagant gift-giving have changed or prices have risen beyond the level that most people can afford during this holiday season. 14. The “suicide wave” that followed the United States stock market crash of October 1929 is more legend than fact. Careful examination of the monthly figures on the causes of death in 1929 shows that the number of suicides in October and in November was comparatively low. In only three other months were the monthly figures lower. During the summer months, when the stock market was flourishing, the number of suicides was substantially higher. Which one of the following, if true, would best challenge the conclusion of the passage? (A) The suicide rate is influenced by many psychological, interpersonal, and societal factors during any given historical period. (B) October and November have almost always had relatively high suicide rates, even during the 1920s and 1930s. (C) The suicide rate in October and November of 1929 was considerably higher than the average for those months during several preceding and following years. (D) During the years surrounding the stock market crash, suicide rates were typically lower at the beginning of any calendar year than toward the end of that year.(C) (E) Because of seasonal differences, the number of suicides in October and November of 1929 would not be expected to be the same as those for other months.15. A well-known sports figure found that combining publicity tours with playing tours led to problems, so she stopped combining the two. She no longer allows bookstore appearances and playing in competition to occur in the same city within the same trip. This week she is traveling to London to play in a major competition, so during her stay in London she will not be making any publicity appearances at any bookstore in London. Which one of the following most closely parallels the reasoning used in the passage? (A) Wherever there is an Acme Bugkiller, many wasps are killed. The Z family garden has an Acme Bugkiller, so any wasps remaining in the garden will soon be killed. (B) The only times that the hospital’s emergency room staff attends to relatively less serious emergencies are times when there is no critical emergency to attend to. On Monday night the emergency room staff attended to a series of fairly minor emergencies, so there must not have been any critical emergencies to take care of at the time. (C) Tomato plants require hot summers to thrive. Farms in the cool summers of country Y probably do not have thriving tomato plants. (D) Higher grades lead to better job opportunities, and studying leads to higher grades. Therefore, studying will lead to better job opportunities.(B) (E) Butter knives are not sharp. Q was not murdered with a sharp blade, so suspect X’s butterknife may have been the murder weapon.Questions 16-17 The advanced technology of ski boots and bindings has brought a dramatic drop in the incidence of injuries that occur on the slopes of ski resorts: from 9 injuries per 1,000 skiers in 1950 to 3 in 1980. As a result, the remainder of ski-related injuries, which includes all injuries occurring on the premises of a ski resort but not on the slopes, rose from 10 percent of all ski-related injuries in 1950 to 25 percent in 1980. The incidence of these injuries, including accidents such as falling down steps, increases with the amount of alcohol consumed per skier.16. Which one of the following can be properly inferred from the passage? (A) As the number of ski injuries that occur on the slopes decreases, the number of injuries that occur on the premises of ski resorts increases. (B) The amount of alcohol consumed per skier increased between 1950 and 1980. (C) The technology of ski boots and bindings affects the incidence of each type of ski-related injury. (D) If the technology of ski boots and bindings continues to advance, the incidence of ski-related injuries will continue to decline.(E) (E) Injuries that occurred on the slopes of ski resorts made up a smaller percentage of ski-related injuries in 1980 than in 1950.17. Which one of the following conflicts with information in the passage? (A) The number of ski injuries that occurred on the slopes was greater in 1980 than in 1950. (B) A skier was less likely to be injured on the slopes in 1950 than in 1980. (C) The reporting of ski injuries became more accurate between 1950 and 1980. (D) The total number of skiers dropped between 1950 and 1980.(B) (E) Some ski-related injuries occurred in 1980 to people who were not skiing.18. Learning how to build a nest plays an important part in the breeding success of birds. For example, Dr. Snow has recorded the success of a number of blackbirds in several successive years. He finds that birds nesting for the first time are less successful in breeding than are older birds, and also less successful than they themselves are a year later. This cannot be a mere matter of size and strength, since blackbirds, like the great majority of birds, are fully grown when they leave the nest. It is difficult to avoid the conclusion that they benefit by their nesting experience. Which one of the following, if true, would most weaken the argument? (A) Blackbirds build better nests than other birds. (B) The capacity of blackbirds to lay viable eggs increases with each successive trial during thefirst few years of reproduction. (C) The breeding success of birds nesting for the second time is greater than that of birds nesting for the first time. (D) Smaller and weaker blackbirds breed just as successfully as bigger and stronger blackbirds. (B) (E) Up to 25 percent of all birds are killed by predators before they start to nest. 来源:考试大-LSAT 19. How do the airlines expect to prevent commercial plane crashes?Studies have shown that pilot error contributes to two-thirds of all such crashes. To address this problem, the airlines have upgraded their training programs by increasing the hours of classroom instruction and emphasizing communication skills in the cockpit. But it is unrealistic to expect such measures to compensate for pilots’ lack of actual flying time. Therefore, the airlines should rethink their training approach to reducing commercial crashes. Which one of the following is an assumption upon which the argument depends? (A) Training programs can eliminate pilot errors. (B) Commercial pilots routinely undergo additional training throughout their careers. (C) The number of airline crashes will decrease if pilot training programs focus on increasing actual flying time. (D) Lack of actual flying time is an important contributor to pilot error in commercial plane crashes.(D) (E) Communication skills are not important to pilot training programs.20. All savings accounts are interest-bearing accounts. The interest from some interest-bearing accounts is tax-free, so there must be some savings accounts that have tax-free interest. Which one of the following arguments is flawed in a way most similar to the way in which the passage is flawed? (A) All artists are intellectuals. Some great photographers are artists. Therefore, some great photographers must be intellectuals. (B) All great photographers are artists. All artists are intellectuals. Therefore, some great photographers must be intellectuals. (C) All great photographers are artists. Some artists are intellectuals. Therefore, some great photographers are intellectuals. (D) All great photographers are artists. Some great photographers are intellectuals. Therefore, some artists must be intellectuals.(C) (E) All great photographers are artists. No artists are intellectuals. Therefore, some great photographers must not be intellectuals.21. One method of dating the emergence of species is to compare the genetic material of relatedspecies. Scientists theorize that the more genetically similar two species are to each other, the more recently they diverged from a common ancestor. After comparing genetic material from giant pandas, red pandas, raccoons, coatis, and all seven bear species, scientists concluded that bears and raccoons diverged 30 to 50 million years ago. They further concluded that red pandas separated from the ancestor of today’s raccoons and coatis a few million years later, some 10 million years before giant pandas diverged from the other bears. Which one of the following can be properly inferred from the passage? (A) Giant pandas and red pandas are more closely related than scientists originally thought they were. (B) Scientists now count the giant panda as the eighth species of bear. (C) It is possible to determine, within a margin of just a few years, the timing of divergence of various species. (D) Scientists have found that giant pandas are more similar genetically to bears than to raccoons. (D) (E) There is substantial consensus among scientists that giant pandas and red pandas are equally related to raccoons.Questions 22-23 Despite improvements in treatment for asthma, the death rate form this disease has doubled during the past decade from its previous rate. Two possible explanations for this increase have been offered. First, the recording of deaths due to asthma has become more widespread and accurate in the past decade than it had been previously. Second, there has been an increase in urban pollution. However, since the rate of deaths due to asthma has increased dramatically even in cities with long-standing, comprehensive medical records and with little or no urban pollution, one must instead conclude that the cause of increased deaths is the use of bronchial inhalers by asthma sufferers to relieve their symptoms.22. Each of the following, if true, provides support to the argument EXCEPT: (A) Urban populations have doubled in the past decade. (B) Records of asthma deaths are as accurate for the past twenty years as for the past ten years. (C) Evidence suggests that bronchial inhalers make the lungs more sensitive to irritation by airborne pollen. (D) By temporarily relieving the symptoms of asthma, inhalers encourage sufferers to avoid more beneficial measures.(A) (E) Ten years ago bronchial inhalers were not available as an asthma treatment.23. Which one of the following is an assumption on which the argument depends?(A) Urban pollution has not doubled in the past decade. (B) Doctors and patients generally ignore the role of allergies in asthma. (C) Bronchial inhalers are unsafe, even when used according to the recommended instructions. (D) The use of bronchial inhalers aggravates other diseases that frequently occur among asthma sufferers and that often lead to fatal outcomes even when the asthma itself does not.(E) (E) Increased urban pollution, improved recording of asthma deaths, and the use of bronchial inhalers are the only possible explanations of the increased death rate due to asthma.24. There is little point in looking to artists for insights into political issues. Most of them hold political views that are less insightful than those of any reasonably well-educated person who is not an artist. Indeed, when taken as a whole, the statements made by artists, including those considered to be great, indicate that artistic talent and political insight are rarely found together. Which one of the following can be inferred from the passage? (A) There are no artists who have insights into political issues. (B) A thorough education in art makes a person reasonably well educated. (C) Every reasonably well-educated person who s not an artist has more insight into political issues than any artist. (D) Politicians rarely have any artistic talent.(E) (E) Some artists are no less politically insightful than some reasonably well-educated persons who are not artists. 来源:考试大-LSAT。
侦探推理智力测试题(3篇)
第1篇背景设定:在一个寒冷的冬夜,一座古老的城堡被冰雪覆盖,内部却发生了一起离奇的谋杀案。
城堡的主人约翰·格雷森先生被发现死在自己的书房内,身边散落着一些书籍和一张地图。
根据调查,案发时间为午夜,但书房的门窗都紧闭,没有明显的外来入侵痕迹。
警方初步判断这是一起谋杀案,但案件疑点重重,需要一位杰出的侦探来解开谜团。
测试题目:第一章:书房之谜1. 约翰·格雷森先生的尸体位于书房中央,呈仰卧状,双手交叉放在胸前,头向后仰,嘴角带着一丝微笑。
2. 约翰先生的眼睛紧闭,但瞳孔已经放大,表明他在临死前可能经历了极大的恐惧。
3. 房间的地板上散落着一些书籍,其中一本是关于古代神秘仪式的书籍,另一本是关于城堡历史的书籍。
4. 约翰先生的西装口袋里有一张地图,上面标注了城堡中一些不为人知的房间和通道。
5. 书桌上有一杯半空的红酒,杯口处有明显的指纹,但并非约翰先生的手指纹。
第二章:书房的线索1. 约翰先生的尸体旁边有一根点燃的蜡烛,蜡烛已经燃尽,但并未完全烧尽。
2. 地毯上有一处明显的脚印,脚印周围有血迹,血迹已经凝固。
3. 书桌上的一本书被翻到了一页,那页内容是关于一个古老的诅咒。
4. 约翰先生的写字台上有一封未寄出的信,信中提到了一位名为艾米丽的女子。
5. 书房内的挂钟显示的时间是11:55,但钟表上的时间似乎被某种力量固定了。
第三章:城堡的秘密1. 城堡内有一位名叫哈里森的管家,他声称案发当晚一直在厨房忙碌,没有离开过。
2. 管家提供了一个证物——一串钥匙,其中一把钥匙可以打开书房的锁。
3. 艾米丽是约翰先生的表妹,她最近才搬来城堡居住,声称是为了照顾生病的约翰先生。
4. 约翰先生生前有一位好友,名叫马克,他最近因为生意失败而负债累累。
5. 城堡内有一处地下密室,据说是为了存放珍贵物品而建造的。
第四章:真相大白1. 侦探在书房的墙壁上发现了一行字:“艾米丽,你的秘密将被揭露。
”2. 侦探在地下密室中发现了一幅画像,画像上的女子正是艾米丽。
推理能力面试题目(3篇)
第1篇一、背景介绍随着社会的发展,人们对于推理能力的需求越来越高。
推理能力不仅是解决问题的关键,更是衡量一个人综合素质的重要标准。
为了选拔具备优秀推理能力的应聘者,某公司特举办一场推理能力面试。
以下为面试题目,请各位应聘者认真阅读并作答。
二、面试题目1. 题目一:逻辑推理题小王、小李、小张、小赵四人在一次比赛中分别获得了第一名、第二名、第三名和第四名。
已知以下信息:(1)小王不是第一名。
(2)小张比小赵成绩好。
(3)小李的名次比小张高。
请根据以上信息,推断出四人的名次顺序。
2. 题目二:分析推理题某市发生一起盗窃案,警方在现场发现以下线索:(1)盗窃现场附近有监控录像,录像显示案发时间为凌晨1点。
(2)被盗物品价值较高,包括一台笔记本电脑、一部手机和一台平板电脑。
(3)被盗现场附近有一名目击者,他声称在案发时看到一个身穿黑色外套、戴着黑色口罩的男子从现场离开。
(4)警方调查发现,嫌疑人甲、乙、丙、丁四人都有作案嫌疑。
请根据以上线索,分析哪位嫌疑人与盗窃案有关,并给出推理过程。
3. 题目三:思维导图题请以“时间管理”为主题,绘制一张思维导图,包含以下内容:(1)时间管理的定义。
(2)时间管理的重要性。
(3)提高时间管理能力的技巧。
4. 题目四:数学推理题某班级有40名学生,其中有20名女生。
已知以下信息:(1)男生平均身高为1.75米。
(2)女生平均身高为1.60米。
(3)男生和女生的身高总和为72米。
请根据以上信息,推断出男生和女生的人数。
5. 题目五:故事推理题某天,甲、乙、丙、丁四人一起去看电影。
电影结束后,他们讨论起了电影中的推理情节。
以下是他们各自的说法:甲:我觉得凶手是乙。
乙:我觉得凶手是丙。
丙:我觉得凶手是丁。
丁:我觉得凶手是甲。
经过调查,他们发现其中只有一个人说的是真话,其他三人都在说谎。
请根据以上信息,推断出谁是凶手。
6. 题目六:情景推理题小明、小红、小华、小丽四人在一次户外活动中迷路了。
2021年美国法学院入学考试LSAT题库和答案
2021年美国法学院入学考试LSAT题库和答案
2021年美国法学院入学考试(LSAT)题库【真题精选+章节题库+模拟试题】
内容简介
本题库包括真题精选、章节题库和模拟试题三部分。
具体如下:
第一部分为真题精选及详解。
根据试卷结构,精选LSAT阅读理解、逻辑推理、分析推理部分典型考试真题,每题均给出答案及解析。
既可以体验真实考试,也可以测试自己的水平。
通过该部分学习,学员可充分了解出题风格,熟知解题思路,从而能够有针对性地备考。
第二部分为章节题库。
按照试卷结构,共分为阅读理解、逻辑推理、分析推理共三种考试题型。
每种题型精选专项练习,帮助学员加深对解题思路的理解和掌握,并在实战中灵活运用。
专项练习是在参考众多相关考试用书、国内外权威杂志以及优秀论文等大量素材的基础上精心设计而成,具有很强的针对性和实用性。
每题均提供答案及解析。
第三部分为模拟试题及详解。
由圣才辅导名师根据历年命题规律及热门考点进行考前预测,可用于考前冲刺或摸底自测。
•
试看部分内容
•第一部分真题精选及详解
•LSAT阅读理解真题精选及详解•LSAT逻辑推理真题精选及详解•LSAT分析推理真题精选及详解•第二部分章节题库
•第1章阅读理解
•第2章逻辑推理(议论)•第3章分析推理(游戏)•第三部分模拟试题
•LSAT模拟试题及详解(一)•LSAT模拟试题及详解(二)。
逻辑推理模拟试题
逻辑推理模拟试题本文将为您提供一组逻辑推理模拟试题,每题均有详细解析。
请您仔细思考并选择正确答案。
题目一:以下是一些关于动物的陈述,请推理出每个动物的名称,每个陈述只适用于一个动物。
1. 这个动物是世界上最大的陆地哺乳动物。
2. 这个动物可以在水中生活。
3. 这个动物的颜色可能是白色、黑色和金色。
4. 这个动物有长长的象牙、褐色的皮肤和大大的耳朵。
题目二:某公司最近组织了一个团队建设活动,共有A、B、C、D、E五位员工参与。
以下是关于这五个员工的陈述,请根据陈述推理出每个员工的职位。
1. A和B是经理。
2. C是秘书。
3. D的职位比A高。
4. E和D是同一级别的。
5. B和C不是同一个职位。
题目三:某班级共有30名学生参与英语考试,以下是一些关于学生成绩的陈述,请根据陈述推理出每个学生的成绩。
1. 有15名学生考试成绩在90分以上。
2. 有5名学生考试成绩在60分以下。
3. 有3名学生考试成绩在60分以上但在90分以下。
4. 有2名学生成绩为满分。
5. Tom的成绩比Jane高,但比John低。
题目四:以下是一些关于某家电产品的陈述,请根据陈述推理出该产品的名称。
1. 这个产品使用电池供电。
2. 这个产品可以连接到其他设备进行音频输出。
3. 这个产品可用于记录声音和必要的文字信息。
4. 这个产品的外观是白色,尺寸适中、易于携带。
题目五:以下是一组数字序列,请根据规律推理出下一个数字是多少。
2, 4, 8, 16, 32, __题目六:以下是一组图案,请根据规律推理出下一个图案是什么。
[图案见附件]解析:题目一的答案是大象。
根据陈述1,世界上最大的陆地哺乳动物是大象;陈述2指出该动物可以在水中生活,对应大象喜欢游泳;陈述3中描述的颜色和陈述4中的特征都与大象相符。
题目二的答案是:A-经理B-秘书C-经理D-经理E-秘书根据陈述1,A和B是经理;陈述2指出C是秘书;根据陈述3,比A职位高的只有D,因此A、B、C都是经理,D与A同级,所以是经理,剩下的E只能是秘书。
LSAT考试全真试题三 含答案(4部分)
LSAT考试全真试题三SECTION 1Time-35 minutes24 QuestionsDirections: Each group of questions in this section is based on a set of conditions. In answering some of the questions, it may be useful to araw a rough diagram. Choose the resoonse that most accurately and completely answers each question and blacken the corresponding space on your answer sheet.Questions 1-6Seven students-fourth-year students Kim and Lee; third-year students Pat and Robin: and second-year students Sandy, Tety and Val-and only those seven, are being assigned a rooms of equal size in a dormitory. Each room assigned must have either one or two or three students assigned to it and will accordingly be called either a single or a double or a triple. The seven students are assigned to moms in accordence with the following conditions:Lio fourth-year student can be assigned to a triple.No second-year student can be assigned to a single.Lee and Pobin must not share the same roomKim and Pat must share the same room.1. Which one of the following is a combination of rooms to which the seven students could be assigned?(A) two triples and one single(B) one triple and four singles(C) three doubles and a stngle(D) two doubles and three singles(E) one double and five singles2. It the room assigned to Robin is a single, which one of the following could be true?(A) There is exactly one double that has a second-year student assigned to it.(B) Lee is assigned to a stngle.(C) Sandy Fat and one other student are zseigned to a triple together.(D) Lixactly three of the rooms assigned to the students are singles(E) Exactly two of the rooms assigned to the students are doubles.3. Which one of the following must be true?(A) Lee is assigned to a single(B) Pat sharts a double with another student(C) Robin shares a double with another student(D) Two of the second-year students share a double with each other(E) Neither of the third-year students is assigned to a single4. If Robin is assigred to a triple, which one of the following must be true?(A) Lee is assigned to a single(B) Two second-year students share a double with each other(C) None of the rooms assigned to the students is a single(D) Two of the rooms assigned to the students are singles.(E) Three of the rooms assigned to the students are singles5. If Terry and Val assigned to different doubles from each other, other, then it must be true of the students rooms that exactly(A) one is a single(B) two are singles(C) two are doubles(D) one is a triple(E) two are triples6. Which one of the following could be true?(A) The two fourth-year students are assigned to singles.(B) The two fourth-year students share a double with cach other.(C) Lee shares a room with a second-year student(D) Lee shares a room with a third-year student(E) Pat shares a triple with two other studentsQuestions 7-11A worker will insert colored light bulbs into a billboard equipped with exactly three light sockets, which are labled lights 1, 2, and 3. The worker has three green bulbs, three purple bulbs, and three yellow bulbs. Seiection of bulbs for the sockets is governed by the following conditions:Whenever light 1 is purple, light 2 must be yellow.Whenever light 2 is purple, light 1 must be green.Whenever light 3 is either purple or yellow, light 2 must be purple.7. Which one of the following could be an accurate list of the colors of light bulbs selected for lights 1, 2 and 3, respectively?(A) green, green, yellow(B) purple, green, green(C) purple, purple, green(D) yellow, purple, green(E) yellow, yellow, yellow8. If light 1 is yellow, then any of the following can be true, EXCEPT:(A) Light 2 is green.(B) Light 2 is purple(C) Light 3 is green(D) Light 3 is purple(E) Light 3 is yellow9. There is exactly one possible color sequence of the three lights if which one of the following is true?(A) Light 1 is purple.(B) Light 2 is purple.(C) Light 2 is yellow(D) Light 3 is purple.(E) Light 3 is yellow10. If no green bulbs are selected, there are exactly how many possible different color sequences of the three lights?(A) one(B) two(C) three(D) four(E) five11. If no two lights are assigned light bulbs that are the same color as each other, then which one of the following could be true?(A) Light I is green, and light 2 is purple.(B) Light I is green, and light 2 is yellow.(C) Light I is purple, and light 2 is yellow.(D) Light I is yellow, and light 2 is green.(E) Light I is yellow, and light 2 is purple.SECTION IITime—35 minutes25 QuestionsDirections: The questions in this section are based on the reasoning contained in brief statements or passages. For some questions. more than one of the choices could conceivably answer the question. However, you are to choose the best answer, that is, the response that most accurately and completely answers the questions. You should not make assumptions that are by blacken the corresponding space on your answer sheet.1. When politicians resort to personal atacks many editortalists criticize thest attacks but most voters pay them scant attention. Eeveryone knows such attacks will end after election day, and politicians can be excused for mudslinging. Political commentators, however, cannot be. Political commentators should be engaged in sustained and senous debate about ideas and policies. In such a context personal attacks on opponents serve not to beat those opponents but to cut off the debate.Which of the following most accurately states the main point of the argument?(A) Dersonal attacks on opponets serve a usuful purpose for politicians.(B) Political commentators should not resort to personal attacks on their opponents.(C) Editonalists are right to criticize politicians who resort to personal attacks on their opponents.(D) The purpose of serious debate about ideas and policies is to counteract the effect of personal attacks by politicians.(E) Voters should be concerned about the personal attacks politicians make on each other.2. Throughout the Popoya Islands community pressure is exerted on people who win the national lottery to share their good fortune with their neighbors. When people living in rural areas win the lottery they invariably throw elaborate neighborhood feasts, often wiping, out all of their lottery winmmings. However, in the cities, lottery winners frequently use their winnings for their own personal investment rather than sharing their good fortune with their neighbors.Which one of the following true, contributes most to an explanation of the difference between the behavior of lottery winners in rural areas and those in cities?(A) Twice as many Popoyans live in rural areas as live in the city.(B) Popoyan city dwellers tend to buy several lottery tickets at a time, but they buy tickets less frequently than do rural dwellers.(C) Lottery winners in rural areas are notified of winning by public posting of lists of winners, but notification in the city is by private mail.(D) Families in rural areas in the Popoyas may contain twelve or foruteen people, but city families average six or seven.(E) Twice as many lottery tickets are sold in rural areas as are sold in the city.3. A new medication for migraine seems effective, but there is concern that the medication might exacerbate heart disease. If patiens with heart disease take the medication under careful medical supervision. however, harmful side effects can definitely be averted. The concern about those side effects is thus unfounded.The argument depends on which one of the following assumptions?(A) The new medication actually is effective when taken by patients with heart disease.(B) No migraine sufferers with heart disease will take the new medication except under careful medical supervision.(C) Most migraine sufferers who have taken the new medication in trials also had heart disease(D) The new medication has various other side effects, but none as serious as that of exacerbating heart disease.(E) The new medication will displace all migrame medicztions currently being used.4. The highest-ranking detectives in the city s police department are also the most adept at solving crimes. Yet in each of the past ten years. the average success rate for the city s highest-ranking detectives in solving crimnal cases has been no higher than the average success rate for its lowest-ranking detectives.Which one of the follwing, if true, most helps to resolve the apparent paradox?(A) The detectives who have the highest success rate in solving criminal cases are those who have worked as detectives the longest.(B) It generally takes at least ten years for a detective to rise from the lowest to the highest ranks of the city s detective force.(C) Those detectives in the police department who are the most adept at solving criminal cases are also those most likely to remain in the police department.(D) The police department generally gives the criminal cases that it expects to be the easiest to solve to its lowest-ranking detectives.(E) None of the lowest-ranking detectivesin the police department had experiecne in solving critninal cases prior to joining the police deparment.5. Imgation runoff from neighboring farms may well have increased the concentration of phosphorus in the local swamp above previous levels, but the claim that the increase in phosphorus is harming the swamp s native aquatie wildlife is false: the phospborus concentration in the swamp is actually less than that found in certain kinds of bottled water that some peopledrink every day.The argument is vulnerable to criticism on the ground that it(A) makes exaggerations in formulating the claim against which it argues(B) bases its conclusion on two contradictiry claims(C) relies on evidence the relevance of which has not been established(D) concedes the very point that it argues against(E) makes a generalization that is unwarranted because the sources of the data on which it is based have not been specified.6. Copyright laws protect the rights of writers to profits earned from their writings. whereas patent laws protec: inventors rights to profits earned from their inventions In jawade, when computer-software writers demanded that their rights to profit be protected, the courts determined that information written for a machine does not fit into either the copyright or the patent category. Clearly, therefore, the profit rights of computer-software writers remain unprotected in Jawade.Which one of the following is an assumption on which the argument depends?(A) Computer-software writers are not an influential enough group in Jawade for the government to consider modifying existing copyright laws in order to protect this group s profit rights.(B) No laws exist, other than copyright laws and patent laws, that would protect the profit rights of computer-software writers in Jawade.(C) Most of the computer software used in Jawade is imported from other countries.(D) Computer software is more similar to writings covered by copyright laws than it is to inventions covered by patent laws.(E) Copyright laws and patent laws in Jawade have not been modified since their original adoption.7. Brownlea s post office must be replaced with a larger one. The present one cannot be expanded. land near the present location in the center of town is more expensive than land on the outskirts of town. Since the cost of acquiring a site is a significant part of the total construction cost, the post office clearly could be built more cheaply on the outskirts of town.Which one of the following, if true, most seriously undermines the argument s stated conclusion?(A) The new post office will have to be built in accordance with a demanding new citywide building code.(B) If the new post office is built on the outskirts of town, it will require a parking lot, but if sited near the present post office it will not.(C) If the new post office is built on the outskirts of town, current city bus routes will have to be expanded to provide access.(D) If the new post office is built on the outskirts of town, residents will make decreased use of post office boxes, with the result that mail carriers will have to deliver more mail to homes.(E) If the new post office is built near the center of town, disruptions to city traffic would have to be minimized by taking such steps as doing some construction work in stages at night and on weekends.8. in the past, the railroads in Ostronia were run as regional monopelies and opeerated with little regard for what customers wanted. In recent years, with improvements to the Ostronian national highway network the railroad companies have faced heavy competition from longdistance trucking companies. But because of government subsidies that have permitted Ostronian railroad companies to operate even while incuring substantial losses, the companies continie to disregard customers needs and desires.If the statements above are true, which one of the following must also be true on the basis of them?(A) If the government of Ostronia ceases to subsidize railroad companies. few of those companies will continue to operate.(B) Few companies in Ostronia that have received subsidies from the government have taken the needs and desires of their customers into account.(C) Without government subsidies, railroad companies in Ostronia would have to increase the prices they charge their customers.(D) The transportation system in Ostronia is no more efficient today than it was in the past.(E) In recent years, some companies in Ostronia that have had little regard for the desires of their customers have nonetheless survived.9. Although Damon had ample time carlier in the month to complete the paper he is scheduled to present at a professional conference tomorrow morning he repeatedly put off doing it. Damon could still get the paper ready in time, but only if he works on it all evening without interruption. However, his seven-year-old daughter s tap-dance recital takes place this evening and Damon had promised both to attend and to take his daughter and her friends out for ice cream afterward. Thus, because of his procrastination. Damon will be forced to choose between his professional and his farmily responsibilities.The argument proceeds by(A) providing evidence that one event will occur in order to establish that an altemative event cannot occur(B) showing that two situations are similar in order to justify the claim that someone with certain responsibilities in the first situation has similar responsibilities in the second situation(C) invoking sympathy for someone who finds himself in a dilemma in order to excuse that person s failure to meet all of his responsibilities(D) making clear the extent to which someone s actions resulted in harm to others in order to support the claim that those actions were irresponsible(E) demonstrating that two situations cannot both occur by showing that something necessary for one of those situations is incompatible with something necessary for the other situation10. The increase in the price of jet fuel is due to a sharp decrease over the past year in the supply of jet fuel available relative to demand. Nonetheless, the amount of jet fuel available for sale is larger today than it was last year.If the statements above are true, which one of the following conclusions can be properly drawn on the basis of them?(A) The demand for jet fuel has increased over the past year.(B) The fuel efficiency of jet engines has increased over the past year.(C) The number of jet airline flights has decreased over the past year.(D) The cost of refining petroleum for jet fuel has increased over the past year.(E) The supply of petroleum available for jet fuel ha decreased over the past year.SECTION IIITime—35 minutes25 QuestionsDirections: The questions in this section are based on the reasoning contained in brief statements or passages. For some questions, more than one of the choices could conceivably answer the question. However, you are to choose the best answer, that is, the response that most accurately and completely answers the question. You should not make assumptions that are by commonsense standards implausible, superfluous, or incompatible with the passage. After you have chosen the best answer, blacken the corresponding space on your answer sheet.1. Everyone sitting in the waiting room of the school s athletic office this morning at nine o clock had just registered for a beginners tennis clinic. John, Mary, and Teresa were all sitting in the waiting room this morning at nice o clock. No accomplished tennis player would register for a beginners tennis clinic.If the statements above are true, which one of the following must also be true on the basis of them?(A) None of the people sitting in the school s athletic office this morning at nine o clock had ever played tennis.(B) Everyone sitting in the school s athletic office this morning at nine o clock registered only for a beginners tennis clinic.(C) John, Mary, and Teresa were the only people who registered for a beginners tennis clinic this morning.(D) John, Mary, and Teresa were the only people sitting in the waiting room of the school s athletic office this morning at nine o clock(E) Neither John nor Teresa is an accomplished tennis player.2. Most people who ride bicycles for pleasure do not ride until the warm weather of spring and summer arrives. Yet it is probably more effective to advertise bicycles earlier in the year. Most bicycles are purchased in the spring, but once shoppers are ready to shop for a bicycle, they usually have already decided which brand and model of bicycle they will purchase. By then it is generally too late to induce them to change their minds.The main point of the argument is that(A) bicycle advertisements are probably more effective if they appear before the arrival of warm spring weather(B) most bicycle purchasers decide on the brand and model of bicycle that they will buy before beginning to shop for a bicycle(C) more bicycles are purchassed in the spring than at any other time of year.(D) in general, once a bicycle purchaser has decided which bicycle he or she intends to purchase, it is difficult to bring about a change in that decision(E) spring and summer are the time of year in which bicycle riding as a leisure activity is most popular3. During 1991 the number of people in the town of Bayburg who received municipal food assistance doubled, even though the number of people in Bayburg whose incomes were low enough to qualify for such assistance remained unchanged.Which one of the following, if true, most helps to resove the apparent discrepancy in the information above?(A) In 1990 the Bayburg Town Council debated whether or not to alter the eligibility requirements for the food assistance program but ultimately decided not to change them.(B) In 1990 the Bayburg social service department estimated the number of people in Bayburg who might be eligible for the food assistance program and then informed the Bayburg Town Council of the total amount of assistance likely to be needed.(C) During 1991 many residents of a nearby city lost their jobs and moved to Bayburg in search of work.(D) During 1991 the number of applicants for food assistance in Bayburg who were rejected on the basis that their incomes were above the maximum allowable limit was approximately the same as it had been in 1990.(E) During 1991 Bayburg s program of rent assistance for low-income tenants advertised widely and then informed all applicants about other assistance programs for which they would be qualified.4. Campaigning for election to provincial or state office frequently requires that a candidate spend much time and energy catering to the interests of national party officials who can help the candidate to win office. The elected officials who campaign for reelection while they are in office thus often fail to serve the interests of their local constituencies.Which one of the following is an assumption made-by the argument?(A) Catering to the interests of national party officials sometimes conflicts with serving the interests of a provincial or state official s local constituencies.(B) Only by catering to the interests of national party officials can those who hold provincial or state office win reelection.(C) The interests of iocal constituencies are well served only by elected officials who do not cater to the interests of national party officials.(D) Officials elected to provincial or state office are obligated to serve only the interests of constituents who beling to the same party as do the officials.(E) All elected officials are likely to seek reelection to those offices that are not limited to one term.5. Since Professor Smythe has been head of the deparment the most distinguished member of the faculty has resigned, fewer new courses have been developed, student has dropped, and the reputation of the department has gone down. These facts provide conclusive evidence that Professor Smythe was appointed to undermine the department.The reasoning in the argument is flawed because the argumetn(A) overlooks the fact that something can have the reputation for being of poor quality without being of poor quality(B) bases a general claim on a few exceptional instances(C) assumes that because an action was followed by a change, the action was undertaken to bring about that change.(D) fails to distinguish between a decline in quantity and a decline in quality(E) presupposes what it purports to establish6. Books about architectural works. unless they are not intended for a general audience, ought to include discussions of both the utility and the aesthetic appeal of each of the buildings they consider. If they do not, they are flawed. Morton s book on Italian Baroque palaces describes these palaces functional aspects, but fails to mention that the main hall of a palace he discusses at length has a ceiling that is one of the truly breathtaking masterpieces of Western art.If the statements above are true, it would be necessary to establish which one of the following in order to conclude that Morton s book is flawed?(A) Morton s deseription of the palaces utility is inaccurate(B) Morton s book does not discuss aspects of the palaces other than utility and aesthetic appeal(C) Morton s book is intended for a general audience.(D) The passage discussing the palace plays a very important role in helping to establish the overall argument of Morton s book.(E) The palace discussed at length is one of the most aesthetically important of those treated in Morton s book.7. Of all the photographs taken of him at his wedding there was one that John and his friends sharply disagreed about. His friends all said that this particular picture did not much resemble him, but John said that on the contrary it was the only photograph that did.Which one of the following, if true about the photograph most helps to explain John s disagreement with his friends?(A) It, unlike the other photographs of John, showed him in the style of dress he and his friends usually wear rather than the formal clothes he wore at the ceremony.(B) It was the only photograph taken of John at his wedding for which the photographer had used a flash.(C) It was a black-and-white photograph, whereas the other photographs that showed John were mostly color photographs.(D) It was unique in showing John s face reflected in a mirror, the photographer having taken the photograph over John s shoulder.(E) It was one of only a few taken at the wedding that showed no one but John.Questions 8-9Eva: A "smart highway" system should be installed, one that would monitor areawide traffic patterns and communicate with computers in vehicles or with programmable highway signs to give drivers information about traffic congestion and alternate routes. Such a system, we can infer, would result in improved traffic flow in and around cities that would do more than improve drivers tempers; it would decrease the considerable loss of money and productivity that now results from traffic congestion.Lines: There are already traffic reports on the radio. Why would a "smart highway" system be any better?8. Eva s argument depends on the assumption that(A) on "smart highways" there would not be the breakdowns of vehicles that currently cause traffic congestion(B) traffic lights, if coordinated by the system, would assure a free flow of traffic(C) traffic flow in and around cities is not now so congested that significant improvement is impossible(D) the type of equipment used in "smart highway" systems would vary from one city to another(E) older wehicles could not be fitted with equipment to receive signals sent by a "smart highway" system9. If Eva responded to Luis by saying that the current one-minute radio reports are too short to give a sufficient description of overall patterns of traffic congestion, which one of the following, if true, would most strengthen Luis s challenge?(A) Bad weather, which radio stations report, would cause traffic to slow down whether or not a "smart highway" system was in operation.(B) It would be less costly to have radio stations that give continual, lengthier traffic reports than to install a "smart highway" system.(C) Radio reports can take note of congestion once it occurs, but a "smart highway" system could anticipate and forestall it in many instances.(D) The proposed traffic monitoring would not reduce the privacy of drivers.(E) Toll collection booths, which constiture traffic bottlenecks, would largely be replaced in the "smart highway" system by electronic debiting of commuters accounts while traffic proceeded at full speed.10. The terms "sex" and "gender" are often used interchangeably. But "sex" more properly refers to biological differences of male and female, while "gender" refers to society s construction of a system that identifies what is masculine and feminine. Unlike the set of character isties defining biological sex, the set of traits that are associated with gender does not sort people into two nonoverlapping groups. The traits character ize people in a complex way, so that a person may have both "masculine" and "feminine" traits.Which one of the following statements best expresses a main point of the argument?(A) Distinctions based on gender are frequently arbitrary.(B) Gender traits are not determined at birth.(C) Masculine gender traits are highly correlated with maleness.(D) The terms "sex" and "gender" are not properly interchangeable.(E) Society rather than the individual decides what is considered proper behavior.11. Raising the tax rate on essential goods—a traditional means of increasing govemment revenues—invariably turns low-and middle-income taxpayers against the government. Hence government officials have proposed adding a new tax on pruchases of luxury items such as yachts, private planes, jewels, and furs. The officials in government revenues while affecting only the wealthy individuals and corporations who can afford to purchase such items.The answer to which one of the following questions would be most relevant in evaluating the accuracy of the government officials prediction?。
2022年GCT考试模拟试卷第三部分:逻辑推理能力测试
2022年GCT考试模拟试卷第三部分:逻辑推理能力测试2022年GCT考试模拟试卷第三部分:逻辑推理能力测试1.任何人都没有吃过雅各岛上的任何水果,所以无法知道雅各岛上任何水果的口味。
为了合乎逻辑的推出上述结论,需要假设下面哪项为前提?[2分]A.如果一种水果有人品尝过,就可以知道其口味。
(充分条件,不是前提)B.只凭某些人的品尝无法真正知道某种水果的口味。
C.要知道某种水果的口味,需要有人去品尝。
D.人们是通过嗅觉来确定水果口味的。
2.最近的一项研究指出:“经常吃沙棘果对儿童的智力发育有益。
”研究人员对560名儿童进行调查,发现那些经常吃沙棘果的儿童,其智力水平较很少吃沙棘果的儿童要高。
因此,研究人员发现了沙棘果与儿童智力发育之间的联系。
以下哪项如果为真,最不可能削弱上述论证?[2分]A.对成年人的研究发现,每天吃沙棘果的人智力水平并不比很少吃沙棘果人的高。
B.调查显示:沙棘果价格非常高,只有富裕家庭的儿童才经常吃,同时这些家庭有条件实现儿童的早期智力开发。
C.这项儿童发育研究的课题负责人是沙棘果生产商,其目的就是要扩展沙棘果的销售渠道。
D.沙棘果是儿童喜欢的食品,家长经常把沙棘果作为礼物奖给智力表现优异的孩子。
3.凡金属都是导电的。
铜是导电的,所以铜是金属。
下面哪项与上述推理结构最相似?[2分]A.所有的鸟都是卵生动物,蝙蝠不是卵生动物,所以,蝙蝠不是鸟。
B.所有的鸟都是卵生动物,天鹅是鸟,所以天鹅是卵生动物。
C.所有从事工商管理的都要学习企业管理,老陈是学习企业管理的,所以,老陈是从事工商管理工作的。
D.华山险于黄山,黄山险于泰山,所以华山险于泰山。
4.为了有助于人们选择最满意的城市居住,有关部门实施了一项评选“最舒适城市”的活动。
方法是,选择十个方面,包括社会治安、商业设施、清洁程度、绿化程度、教育设施、旅游文化景点等等,每个方面按实际质量的高低,评以1分至10分之间的某一分值,然后求得十个分值的平均数即是这个城市的舒适性指数。
2020年新高考III卷数学逻辑推理题及答案
2020年新高考III卷数学逻辑推理题及答案1. 题目分析与答案解析第一题:以下是一组数字序列: 1, 3, 6, 10, 15, 21...请问下一个数字是多少?解析:从第一项开始,每一项都比前一项多1,所以下一个数字是21 + 6 = 27。
答案:27第二题:某商场正在进行打折促销活动,折扣力度为7折(即商品价格打7折),购物满200元再减40元。
小明购买了一部手机,原价300元。
请问他实际需要支付多少钱?解析:首先,将商品价格打7折:300元 * 0.7 = 210元。
接着,考虑满200元再减40元的优惠。
由于小明购买的商品价格并没达到200元,所以无法再享受这个优惠。
因此,小明需要支付的金额是210元。
答案:210元第三题:某书店正在进行促销活动,原价为160元的教材打8折,折上折,再减30元。
小红购买了这本教材,请问她实际需要支付多少钱?解析:首先,将教材原价打8折:160元 * 0.8 = 128元。
接着,考虑再减30元的优惠。
小红可以享受折上折的优惠,所以需要使用优惠后的价格来计算。
128元 - 30元 = 98元。
因此,小红需要支付的金额是98元。
答案:98元2. 数学逻辑推理题讨论本卷共有三道数学逻辑推理题,涉及到计算和推论等方面的技能。
题目的答案解析已经给出,并且给出了具体计算过程,使读者能够理解和掌握解题方法。
数学逻辑推理题在高考中占有重要的一部分,考察学生的数学思维能力和逻辑推理能力。
通过做这些题目,可以培养学生的思维灵活性和解决问题的能力,同时也能提高他们的数学水平。
3. 结语通过解析2020年新高考III卷数学逻辑推理题,我们可以看到这一类题目涉及到数学计算和逻辑推理,需要学生掌握一定的数学知识和解题技巧。
希望本文的分析能对读者有所帮助,提高他们在数学逻辑推理题上的应试能力。
LSAT逻辑推理及真题解析
考试考查能力
考试考查能力
逻辑推理试题共有两个部分,每部分有24~26道试题。一般每道题都有一篇小的短文或对话,然后针对此短 文或对话提出问题。短文或对话涉及的范围很广,包括哲学、文学、政治、科技、艺术、历史、体育等等。逻辑 推理试题主要测试考生的以下能力:
确定中心思想 找出推理中的假设 从已知事实或前提得出合理结论 确定推理的准则并将之应用于新的论证 确定推理的方法或结构 找出推理的错误及误解 确定新的事实或论证对现有论证或结论的加强或削弱 对论证进行分析 LSAT逻辑推理及真题解析 -聚焦LSAT逻辑推理题
LSAT逻辑推理及真题解析
管卫东主编的图书
01 作者简介
目录
02 逻辑推理
03 考试考查能力
04 逻辑推理题
05 考试逻辑题型
基本信息
《LSAT逻辑推理及真题解析》是管卫东主编的图书。
作者简介
作者简介
《LSAT逻辑推理及真题解析》,网上简称GWD,以其名字命名的GWD题几乎被中国GMAT考生奉为“GMAT真题 集”,他创办了专注于GMAT(及LSAT)培训和北美顶尖商学院留学申请咨询服务的博智教育,担任首席 GMAT/LSAT讲师;作为中国第一个透彻研究并主讲GMAT、LSAT、SAT和GRE这四大思维能力类考试的专家,管卫东 探索出一套独特的逻辑思维体系并结合数学与计算机本科背景,研发了一套专门针对这四大考试的FMA教学体系, 集合英语快速阅读能力提升、客观逻辑思维体系建立和计算机自适应性模拟考试机制,帮助考生短期内快速达到 主考方高分要求。作为著名的应试辅导专家,同时担任高等教育出版社、山东教育电视台、北大附中网校特聘高 考辅导教师,多家媒体教育版块的专栏作家。可见,《LSAT逻辑推理及真题解析》这本书具有很大的参考价值。
fbi的智商测试题(3篇)
第1篇亲爱的参与者,欢迎来到FBI智力挑战测试!这是一场旨在考验您思维深度和广度的挑战。
通过完成以下测试,您将有机会了解自己的智力水平,同时也能体验到FBI特工在日常工作中所面临的思维难题。
请注意,本测试包含多种题型,包括逻辑推理、数学计算、心理测试等,共计2500字以上。
请您认真作答,挑战您的智慧极限!一、逻辑推理题(共10题,每题10分)1. 一家超市的货架上有5瓶牛奶,每瓶牛奶上都有一个标签,分别是A、B、C、D、E。
已知每瓶牛奶的标签都是错误的,请问哪瓶牛奶的标签是正确的?2. 小明、小红、小华、小李、小张五个人参加了一场比赛,比赛结果如下:(1)小明不是第一名;(2)小红不是第二名;(3)小华不是第三名;(4)小李不是第四名;(5)小张不是第五名。
请问,谁是第一名?3. 一个房间里有5个开关,分别控制着房间里的5盏灯。
你只能进房间一次,如何确定哪个开关控制哪盏灯?4. 小明、小华、小丽、小张、小王五个人去电影院看电影,电影院的座位号分别是1、2、3、4、5。
已知:(1)小明坐在小华的左边;(2)小丽坐在小张的右边;(3)小王坐在小明的后面。
请问,小华坐在哪个座位?5. 有5个不同颜色的球,分别是红、黄、蓝、绿、白。
将这5个球依次放入5个不同的盒子中,每个盒子只能放一个球。
已知:(1)红色的球不能放在红色的盒子里;(2)黄色的球不能放在黄色的盒子里;(3)蓝色的球不能放在蓝色的盒子里。
请问,绿色的球应该放在哪个盒子里?6. 一个房间里有5个开关,分别控制着房间里的5盏灯。
你只能进房间一次,如何确定哪个开关控制哪盏灯?7. 小明、小红、小华、小李、小张五个人参加了一场比赛,比赛结果如下:(1)小明不是第一名;(2)小红不是第二名;(3)小华不是第三名;(4)小李不是第四名;(5)小张不是第五名。
请问,谁是第一名?8. 有5个不同颜色的球,分别是红、黄、蓝、绿、白。
将这5个球依次放入5个不同的盒子中,每个盒子只能放一个球。
公务员行政测试逻辑推理思维训练
公务员行政测试逻辑推理思维训练(一)把近年来我国公务员考试的逻辑推理部分与GCT、MBA入学考试的逻辑部分相比较,可以发现他们有很大的相似之处。
而此类题目,尤其是对于MBA(工商管理硕士)、GCT-ME(工程硕士)入学考试的逻辑部分而言,更是主要借鉴美国研究生入学学业能力潜力测试的GMAT、GR、LSAT的逻辑部分。
所以,我们可以从某种程度上说,从某种程度上我国的公务考试的逻辑推理部分,也主要借鉴了ETS所推出的GMA T、GR、LSAT三大研究生入学考试的逻辑部分。
例如GMAT的一道真题:Crops can be traded on the futures market before they are harvested if a poor corn harvest is p redicted prices of corn futures rise; if a bountiful corn harvest is predicted prices of corn futures f all This morning meteorologists are predicting much-needed rain for the corn-growing region starti ng tomorrow. Therefore, since adequate moisture is essential for the current crop s survival price s of corn futures will fall sharply today.Which of the following, if true, most weakens the argument above? ( D )(A) Corn that does not receive adequate moisture during its critical pollination stage will not prod uce a bountiful harvest.(B) Futures prices for corn have been fluctuating more dramatically this season than last season(C) The rain that meteorologists predicted for tomorrow is expected to extend well beyond the co rn-growing region.(D) Agriculture experts announced today that a disease that has devastated some of the corn cr op will spread widely before the end of the growing season.(E) Most people who trade in corn futures rarely take physical possession of the corn they trade.我们再看2005年全国MBA联考的逻辑试题第26题:在期货市场上,粮食在收获前就“出售”。
逻辑分析能力测试题
逻辑分析能力测试题在当今复杂多变的社会环境中,逻辑思维和分析能力成为人们成功的重要因素之一。
而逻辑分析能力测试题则是评估个人在逻辑思维和分析方面的能力的一种方法。
本文将分享一系列逻辑分析能力测试题,旨在提供给读者一个锻炼和检验逻辑思维能力的机会。
第一题:在一个晴朗的早晨,Tom、Jack和Peter三位朋友决定去海边度假。
根据规定,每个人需要带一个物品。
以下是他们带的物品清单:- Tom带了帐篷和食品。
- Jack带了一桶水和草坪椅。
- Peter带了钓具和食品。
请问,为了顺利度假,每个人分别带了哪些物品?解析:由于每个人只带了一个物品,所以根据已给出的信息可以得出以下结论:- Tom带了帐篷和食品。
- Jack带了一桶水和草坪椅。
- Peter带了钓具和食品。
第二题:在一场比赛中,运动员A和B参加了6个项目的角逐,包括游泳、跑步、射箭、举重、跳远和自行车。
以下是他们在各项目中的成绩排名:- 运动员A在游泳、跑步、射箭、举重、跳远和自行车中的排名分别为第2名、第3名、第1名、第5名、第6名和第4名。
- 运动员B在游泳、跑步、射箭、举重、跳远和自行车中的排名分别为第3名、第1名、第6名、第2名、第4名和第5名。
请问,运动员A和B在比赛中的最终名次分别是多少?解析:根据已给出的信息,我们可以得出以下结论:- 运动员A在比赛中的最终名次为:3 + 1 + 5 + 2 + 6 + 4 = 21。
- 运动员B在比赛中的最终名次为:2 + 3 + 1 + 5 + 6 + 4 = 21。
可见,运动员A和B在比赛中的最终名次均为21名。
第三题:有五个国家的男子篮球队参加一项比赛,最终的排名已经确定。
以下是五个国家队在比赛中的名次:- 美国队排名第1。
- 德国队排名第3。
- 中国队排名第5。
- 巴西队排名第4。
请问,排名第2的国家队是哪一个?解析:根据题目已给出的信息,我们可以得出以下结论:- 美国队排名第1。
- 德国队排名第3。
2023年LSAT考试真题
2023年LSAT考试真题2023年LSAT考试真题已经公布,这是一次备受关注的考试。
LSAT(美国法学入学考试)是许多想要在美国攻读法学硕士(JD)学位的学生必须参加的考试。
在这个真题里,我们将探讨一些题目类型和例题。
希望以下内容对你有所帮助。
题目一:逻辑推理逻辑推理是LSAT考试中的重要部分。
在这一部分,学生需要根据一些给定的条件进行分析和推理,以确定哪个选项是最符合逻辑的。
以下是一个例题:"如果马丁是个音乐家,那么他会擅长吉他。
马丁不擅长吉他。
所以,他不是音乐家。
"根据上述陈述,学生需要选出以下哪个选项是最恰当的:A. 马丁是个音乐家,同时也擅长吉他。
B. 马丁是个音乐家,但并不擅长吉他。
C. 马丁不是音乐家,但擅长吉他。
D. 马丁不是音乐家,同时也不擅长吉他。
通过推理,我们可以得出答案是D。
因为题目中给出的条件是,如果马丁是音乐家,那么他会擅长吉他。
而题目中明确告诉我们,他不擅长吉他,因此可以判断他不是音乐家。
题目二:阅读理解阅读理解是LSAT考试的另一个重要部分。
在这一部分,学生需要阅读一篇文章,然后回答相关问题。
以下是一个例题:阅读下面这段短文,并回答问题。
"根据最新的研究,饮食和健康之间存在紧密的关联。
科学家们发现,均衡的饮食可以提供人体所需的营养物质,并降低患病风险。
然而,目前的问题在于,很多人的饮食习惯不健康,充斥着高糖、高盐和高脂肪的食物。
为了改善这种情况,人们应该学会选择更健康的食物,并培养良好的饮食习惯。
"问题:根据短文,为什么人们应该选择更健康的食物?通过阅读短文,我们可以看出,科学家们发现均衡的饮食可以提供人体所需的营养物质,并降低患病风险。
因此,人们应该选择更健康的食物来改善他们的饮食习惯。
题目三:逻辑游戏逻辑游戏是LSAT考试中的另一个题型。
这种题型要求学生通过解决一些逻辑问题和条件制约,来确定最终的结果。
以下是一个例题:四名游戏选手Dave,Jane,Mark和Sue,根据一些条件,你需要确定每个人的游戏顺序。
LSAT分析推理备考:LSAT分析推理样题
LSAT分析推理备考:LSAT分析推理样题参加LSAT考试的考生朋友们有福利了,本站编辑为参加LSAT的考生朋友们搜集整理来了LSAT分析推理样题,赶紧查阅练习备考吧~本节中的每一组问题都基于一组条件,在回答一些问题时,画一个粗略的图是有用的。
选择反应最准确、完整地回答问题和表面相应的空间在你的答题纸。
Passage for Question 1A university library budget committee must reduce exactly five of eight areas of expenditure—G, L, M, N, P, R, S, and W—in accordance with the following conditions:If both G and S are reduced, W is also reduced.If N is reduced, neither R nor S is reduced.If P is reduced, L is not reduced.Of the three areas L, M, and R, exactly two are reduced.Question 1If both M and R are reduced, which one of the following is a pair of areas neither of which could be reduced?G, LG, NL, NL, PP, SExplanation for Question 1This question concerns a committee’s decision about which five of e ight areas of expenditure to reduce. The question requires you to suppose that M and R are among the areas that are to be reduced, and then to determine which pair of areas could not also be among the five areas that are reduced.The fourth condition given in the passage on which this question is based requires that exactly two of M, R, and L are reduced. Since the question asks us to suppose that both M and R are reduced, we know that L must not be reduced:Reduced: M, RNot reduced: LThe second condition requires that if N is reduced, neither R nor S is reduced. So N and R cannot both be reduced. Here, since R is reduced, we know that N cannot be. Thus, adding this to what we’ve determined so far, we know that L and N are a pair of areas that cannot both be reduced if both M and R are reduced:Reduced: M, RNot reduced: L, NAnswer choice (C) is therefore the correct answer, and you are done.When you are taking the test, if you have determined the correct answer, there is no need to rule out the other answer choices. However, for our purposes in this section, it might be instructive to go over the incorrect answer choices. For this question, each of the incorrect answer choices can be ruled out by finding a possible outcome in which at least one of the two areas listed in that answer choice are reduced. Consider answer choice (A), which lists the pair G and L. We already know that for this question L must be one of the areas that is not reduced, so all we need to consider is whether G can be one of the area s that is reduced. Here’s one such possible outcome:Reduced: M, R, G, S, WIf areas M, R, G, S, W are reduced, then the supposition for the question holds and all of the conditions in the passage are met:M and R are both reduced, as supposed for this question.Both G and S are reduced, and W is also reduced, so the first condition is satisfied.N is not reduced, so the second condition is not relevant.P is not reduced, so the third condition is not relevant.Exactly two of L, M, and R are reduced, so the fourth condition is satisfied.Thus, since G could be reduced without violating the conditions, answer choice (A) can be ruled out. Furthermore, since G appears in the pair listed in answer choice (B), we can also see that (B) is incorrect.Now let’s cons ider answer choice (D), which lists the pair L and P. We already know that for this question L must be one of the areas that is not reduced, so all we need to consider is whether P can be one of the areas that is reduced. Here’s one such possible outcome:Reduced: M, R, P, S, WIf areas M, R, P, S, and W are reduced, then the supposition for the question holds and all of the conditions in the passage are met:M and R are both reduced, as supposed for this question.G is not reduced, so the first condition is not relevant.N is not reduced, so the second condition is not relevant.P is reduced and L is not reduced, so the third condition is satisfied.Exactly two of L, M, and R are reduced, so the fourth condition is satisfied.Thus, since P could be reduced without violating the conditions, answer choice (D) can be ruled out. Furthermore, since P appears in the pair listed in answer choice (E), we can also see that answer choice (E) is incorrect.This question was of moderate difficulty, based on the number of test takers who answered it correctly when it appeared on the LSAT. The most commonly selected incorrect answer choice was response (E).总结:典型的分析推理的问题,你会在最后找到。
美国法学院入学考试LSAT分析推理真题精选及详解【圣才出品】
LSAT分析推理真题精选及详解Directions:Each group of questions in this section is based on a set of conditions. In answering some of the questions, it may be useful to draw a rough diagram. Choose the response that most accurately and completely answers each question and blacken the corresponding space on your answer sheet.A company employee generates a series of five-digit product codes in accordance with the following rules:the codes use the digits 0, 1, 2, 3, and 4, and no others.Each digit occurs exactly once in any code.the second digit has a value exactly twice that of the first digit.the value of the third digit is less than the value of the fifth digit.1. If the last digit of an acceptable product code is 1, it must be true that the(A) first digit is 2(B) second digit is 0(C) third digit is 3(D) fourth digit is 4(E) fourth digit is 0答案:A解析:如果1在最后一位,第三位比1还小只能是0,而由于前两位必须是倍数关系,那么前两位必然是2和4。
LSAT逻辑分析
GRE逻辑分析辅助材料LSAT 逻辑分析 9 sectionsLeo.XuMay 1st, 2001SECTION 1Time-35 minutes23 QuestionsDirections: Each group of questions in this section is based on a set of conditions. In answering some of the questions, it may be useful to draw a rough diagram. Choose the response that most accurately and completely answers each question and blacken the corresponding space on your answer sheet.Questoins 1-6 A man buys three outfits-X, Y, and Z-each of which consists of two articles of clothing. Each of the articles of clothing is either brown gray or navy. At least one of the outfits is made up of two articles different in color from one another. No more than two of the outfits contain the same combination of colors. Outfit X contains at least one navy article of clothing. Outfit X contains at least one brown article of clothing and does not contain a gray article.1. Which one of the following can be the colors of the man's outfits? (A) X: gray and navy; Y: brown and gray; Z: gray and gray (B) X: brown and gray; Y: brown and navy; Z: gray and gray (C) X: navy and navy; Y: brown and brown; Z: brown and navy (D) X: brown and navy; Y: brown and navy; Z: brown and navy (E) X: navy and navy; Y: brown and brown; Z: navy and navy2. If outfits X and Y each consist of one brown article and one navy article of clothing, what combinations for outfit Z? (A) 2 (B) 3 (C) 4 (D) 5 (E) 63. If outfit Z does not contain two brown items of clothing, what is the maximum number of items of clothing in the three outfits that can be navy? (A) 1 (B) 2 (C) 3 (D) 4 (E) 54. If outfit Y consists of two brown articles of clothing and outfit Z consists of two navy items, what is the total number of possible color combinations for outfit X? (A) 1 (B) 2 (C) 3 (D) 4 (E) 55. Which one of the following color combinations for outfit Z would be acceptable under any of the acceptable color combinations for outfits X and Y? (A) gray and gray (B) brown and gray (C) brown and brown (D) brown and navy (E) navy and navy6. If no two outfits contain the same color combination but each contains at least one navy item, which one of the following is a complete and accurate list of the possible combinations for outfit X? (A) gray and navy (B) brown and navy (C) navy and navy (D) gray and navy; brown and navy (E) gray and navy; navy and navyQuestion 7-11 Five seats on a train are arranged in two rows that face each other. The seats in row 1 are seat 1 and seat 2. The seats in row 2 are seat 3, seat 4, and seat 5 in that order. Seat 1 is directly across from seat 3. Seat 2 is directly across from seat 4. Seat 5 is not directly across from any other seat. Three women-, Betty, and Carol-and two men-David Edmund-each must be seated in one of me five seats, one person to a seat. Carol sits next to neither Betty nor David. Alice does not sit directly across from Carol.7. If Edmund sits directly across from Betty, which one of the following must be true? (A) Alice does not sit directly across from anyone. (B) Alice sits directly across from David. (C) Alice sits next to Edmund. (D) David sits next to Alice. (E) David sits next to Edmund.8. If Edmund does not sit directly across from anyone, which one of the following is a complete and accurate list of those (other than Edmund) who cannot sit in seat 1? (A) Betty (B) Carol (C) David (D) Betty and Carol (E) Betty and David9. If none of the five passengers sits directly across from a passenger of the same sex, how many seating arrangements of the five passengers are possible? (A) exactly 0 (B) exactly 1 (C) exactly 2 (D) exactly 3 (E) exactly 410. How many of the seats could be the one Alice selects as her seat? (A) 1 only (B) 2 only (C) 3 only (D) 4 only (E) 5 only11. How many different seating arrangements of the passengers might there be for row 2? (A) 2 (B) 3 (C) 4 (D) 5 (E) 6Question 12-17 Six people-Julio, Kevin, May, Norma, Olivia, and T amio-participate in a track meet. Two of them enter the marathon, two enter the relay, and two enter the sprint. Each participant enters only one event. If Kevin enters the marathon, then both Julio and May enter the relay, and if both Julio and May enter the relay, then Kevin enters the marathon. If Norma does not enter the sprint, then T amio enters the relay. If Olivia enters the relay, then Julio does not enter the relay. Olivia and T amio do not both enter the relay.12. If Kevin enters the marathon, then which one of the following cannot be true? (A) Julio enters the relay and Norma enters the sprint. (B) Olivia enters the marathon and Norma enters the sprint. (C) T amio enters the marathon and Olivia enters the sprint. (D) T amio enters the relay and Olivia enters the sprint. (E) T amio enters the marathon and May enters the sprint.13. If Norma enters the marathon, then which one of the following cannot be true? (A) Both Kevin and Olivia enter the sprint. (B) Kevin enters the relay and Olivia enters the sprint. (C) Kevin enters the marathon and May enters the relay. (D) Julio enters the marathon and May enters the relay. (E) Julio enters the marathon and Kevin enters the relay.14. If both Kevin and Olivia enter the relay, then which one of the following must be true? (B) May enters the marathon. (C) May enters the sprint. (D) T amio enters the sprint. (E) Norma enters the sprint.15. If both Julio and Kevin enter the same event, then which of the following can be true? Ⅰ.Julio enters the relay. Ⅱ. May enters the marathon. Ⅲ. Olivia enters the relay. (A) Ⅰonly (B) Ⅱ only (C) Ⅲ only (D) Ⅰand Ⅱonly (E) Ⅰ,Ⅱ,and Ⅲ16. If Norma and Olivia each enter different events, then which one of the following cannot be true? (A) Kevin and Olivia enter the marathon. (B) Julio and Norma enter the relay. (C) May and Olivia enter the relay. (D) Norma and T amio enter the relay. (E) Olivia enters the sprint and T amio enters the relay.17. If both Julio and Kevin enter the relay, then which one of the following must be false? (A) May and Norma enter the marathon. (B) May and Olivia enter the marathon. (C) Norma and T amio enter the sprint. (D) May and Norma enter the sprint. (E) May enters the sprint and T amio enters marathon.Questions 18-23 Seven lights-J, K, L, M, N, O, and P-have only two possible settings: on or off. They are arranged on a particular electric circuit as follows. If K is on, L is off; if K is off, L is on. J and N cannot both be on. If M is off, either J or N is on; if either J or N is on, M is off. If P is on, L is on. If O is off, N is off; if O is on, N is on. Assume that the circuit is working as designed.18. Any of the following can be true EXCEPT: (A) J and O are both off. (B) K and N are both off. (C) K and P are both on. (D) L and M are both on. (E) N and O are both on.19. If L and O are on, which one of the following must be true? (A) J is and K is off. (B) J is off and N is on. (C) K is off and M is on. (D) K is off and P is on. (E) M is off and P is on.20. Which one of the following can be true? (A) Only J, K, and M are off. (B) Only J, L, and M are off. (C) Only K, M, and O are off. (D) Only L, N, and O are off. (E) Only M, N, and O are off.21. If P is on, what is the maximum number of lights that can be off? (A) 2 (B) 3 (C) 4 (D) 522. What is the minimum number of lights that must be on? (A) 0 (B) 1 (C) 2 (D) 3 (E) 423. If J is on, which one of the following could be a complete and accurate list of the remaining lights that are also on? (A) K, L, and M (B) L, N, and P (C) L, O, and P (D) M, N, and O(E) L and PCDEBB EAEBD CDCED BACBA CCEsection 2Time-35 minutes24 QuestionsDirections: Each group of questions in this section is based on a set of conditions. In answering some of the questions. If may be useful to draw a rough diagram Choose the response that most accurately and completely answers each question and blacken the corresponding space on your answer sheetQuestion 1-6Eight new students-R, S, T, V, W, X, Y, Z-are being divided among exactly three classes-class 1, class 2, and class3. Classes 1 and 2 will gain three new students each: class 3 will gain two new students. The following restrictions apply: R must be added to class 1. S must be added to class 3. Neither S nor W can be added to the same class as Y. V cannot be added to the same class as Z. If T is added to class 1, Z must also be added to class 1.1.Which one of the following is an acceptable assignment of students to the three classes? 1 2 3 (A) R, T, Y V, W, X S, Z (B) R, T, Z S, V, Y W, X (C) R, W, X V, Y, X S, T (D) R, X, Z T, V, Y S, W (E) R, X, Z V, W, Y S, T2.Which one of the following is a complete and accurate list of classes any one of which could be the class to which V is added? (A) class 1 (B) class 3 (C) class 1, class 3 (D) class 2, class 3 (E) class 1, class 2, class 3.3.If X is added to class 1, which one of the following is a student who must be added to class 2? (A) T (B) V (C) W (D) Y (E) Z4.If X is added to class 3, each of the following is a pair of students who can be added to class 1 EXCEPT (A) Y and Z (B) W and Z (C) V and Y (D) V and W (E) T and z5.If T is added to class 3, which one of the following is a student who must be added to class 2? (B) W (C) X (D) Y (E) Z6.Which one of the following must be true? (A) If T and X are added to class 2. V is added to class 3. (B) If V and W are added to class 1. V is added to class 3. (C) If V and W are added to class 1. V is added to class 3. (D) If V and X are added to class 1. V is added to class 3. (E) If Y and Z are added to class 2. V is added to class 2.Question 7-12Four lions-F, G, H, J-and two tigers-K and M-will be assigned to exactly six stalls, one animal per stall. The stalls are arranged as follows: First Row: 1 2 3 Second Row: 4 5 6The only stalls that face each other are stalls 1 and 4, stalls 2 and 5, and stalls 3 and 6. The following conditions apply: The tigers' stalls cannot face each other. A lion must be assigned to stall 1 H must be assigned to stall 6. J must be assigned to a stall numbered one higher than K's stall. K cannot be assigned to the stall that faces H's stall.7. Which one of the following must be true? (A) F is assigned to an even-numbered stall (B) F is assigned to stall 1 (C) J is assigned to stall 2 or else stall 3 (D) J is assigned to stall 3 or else stall 4 (E) K is assigned to stall 2 or else stall 48. Which one of the following could be true? (A) F's stall is numbered one higher than J's stall (B) H's stall faces M's stall (C) J is assigned to stall 4 (D) K's stall faces J'S stall (E) K's stall is in a different row than J's stall9. Which one of the following must be true? (A) A tiger is assigned to stall 2 (B) A tiger is assigned to stall 5 (C) K's stall is in a different row from M's stall (D) Each tiger is assigned to an even-numbered stall (E) Each lion is assigned to a stall that faces a tiger is stall10. If K's stall is in the same row as H's stall which one of the following must be true? (A) F's stall is in the same row as J's stall (B) F is assigned to a lower-numbered stall than G (C) G is assigned to a lower-numbered stall than M (D) G's stall faces H's stall (E) M's stall is in the same row as G's stall11. If J is assigned to stall 3, which one of the following could be true? (A) F is assigned to stall 2 (B) F is assigned to stall 4 (C) G is assigned to stall 1 (D) G is assigned to stall 4 (E) M is assigned to stall 512. Which one of the following must be true (A) A tiger is assigned to stall 2 (B) A tiger is assigned to stall 4 (C) A tiger is assigned to stall 5 (D) A lion is assigned to stall 3Question 13-18On an undeveloped street, a developer will simultaneously build four houses on one side, numbered consecutively 1, 3, 5, and 7, and four houses on the opposite side, numbered consecutively 2, 4, 6, and 8. Houses 2, 4, 6, and 8 will face houses 1, 3, 5, and 7, respectively. Each house will be exactly one of three styles-ranch, split-level or Tudor-according to the following conditions: Adjacent houses are of different styles. No split-level house faces another split-level house. Every ranch house has at least one Tudor house adjacent to it House 3 is a ranch house. House 6 is a split-level house.13. Any of the following could be a Tudor house EXCEPT house (A) 1 (B) 2 (C) 4 (D) 7 (E) 814. If there is one ranch house directly opposite another ranch house, which one of the following could be true? (A) House 8 is a ranch house (B) House 7 is a split-level house. (C) House 4 is a Tudor house. (D) House 2 is a split-level house. (E) House 1 is a ranch house.15. If house 4 is a Tudor house, then it could be true that house (A) 1 is a Tudor house (B) 2 is a Tudor house (C) 5 is a Tudor house (D) 7 is a Tudor house (E) 8 is a Tudor house16. On the street, there could be exactly. (A) one ranch house (B) one Tudor house (C) two Tudor houses (D) four ranch houses (E) five ranch houses17. If no house faces a house of the same style, then it must be true that house (A) 1 is a split-level house (B) 1 is a Tudor house (C) 2 is a ranch house (D) 2 is a split-level house (E) 4 is a Tudor house18. If the condition requiring house 6 to be a split-level house is suspended but all other original conditions remain the same, the any of the following could be an accurate list of the styles of houses 2, 4, 6, and 8, respectively, EXCEPT: (A) ranch, split-level, ranch, Tudor (B) split-level, ranch, Tudor, split-level (C) split-level, Tudor, ranch, split-level (D) Tudor, ranch. Tudor, split-level (E) Tudor split-level, ranch, TudorQuestions 19-24Within a tennis league each of five teams occupies one of five positions, numbered 1 through 5 in order of rank, with number 1 as the highest position. The teams are initially in the order R, J, S, M, L, with R in position 1. T eams change positions only when a lower-positioned team defeats a higher-positioned team. The rules are as follows: Matches are played alternately in odd-position rounds and in even-position rounds. In an odd-position round, teams in position 3 and 5 play against teams positioned immediately above them. In an even-position round, teams in position 2 and 4 play against teams positioned immediately above them.19. Which one of the of following could be the order of teams, from position 1 through position 5 respectively, after exactly one round of even-position matches if no odd-position round has yet been played? (A) J , R, M, L, S (B) J, R, S, L, M (C) R, J, M, L, S (D) R, J, M, S, L (E) R, S, J, L, M20. If exactly two rounds of matches have been played, beginning with an odd-position round, and if the lower-positioned teams have won every match in those two rounds, then each of the following must be true EXCEPT: (A) L is one position higher than J. (B) R is one position higher than L. (C) S is one position higher than R. (D) J is in position 4. (E) M is position 3.21. Which one of the following could be true after exactly two rounds of matches have been played? (A) J has won two matches. (B) L has lost two matches. (C) R has won two matches. (D) L's only match was played against J. (E) M played against S in two matches.22. If after exactly three rounds of matches M is in position 4, and J and L have won all of their matches, then which one of the following can be true? (A) J is in position 2. (B) J is in position 3. (C) L is in position 2. (D) R is in position 1. (E) S is in position 3.23. If after exactly three rounds M has won three matches and the rankings of the other four teams relative to each other remain the same, then which one of the following must be in position 3? (A) J (B) L (C) M (D) R (E) S24. If after exactly three rounds the teams, in order from first to fifth position, are R, J, L, S, and M, then which one of the following could be the order, from first to fifth position, of the teams after the second round? (A) J, R, M, S, L (B) J, L, S, M, R (C) R, J, S, L, M (D) R, L, M, S, J (E) R, M, L, S, JDEAEC DEBCE CBDBA AEADE ACACsection 3Time-35 minutes24 QuestionsDirections: Each group of questions in this section is based on a set of conditions. In answering some of the questions, it may be useful to araw a rough diagram. Choose the resoonse that most accurately and completely answers each question and blacken the corresponding space on your answer sheet.Questions 1-6 Seven students-fourth-year students Kim and Lee; third-year students Pat and Robin: and second-year students Sandy, T ety and V al-and only those seven, are being assigned a rooms of equal size in a dormitory. Each room assigned must have either one or two or three students assigned to it and will accordingly be called either a single or a double or a triple. The seven students are assigned to moms in accordence with the following conditions: Lio fourth-year student can be assigned to a triple. No second-year student can be assigned to a single. Lee and Pobin must not share the same room Kim and Pat must share the same room.1. Which one of the following is a combination of rooms to which the seven students could be assigned? (A) two triples and one single (C) three doubles and a stngle (D) two doubles and three singles (E) one double and five singles2. It the room assigned to Robin is a single, which one of the following could be true? (A) There is exactly one double that has a second-year student assigned to it. (B) Lee is assigned to a stngle. (C) Sandy Fat and one other student are zseigned to a triple together. (D) Lixactly three of the rooms assigned to the students are singles (E) Exactly two of the rooms assigned to the students are doubles.3. Which one of the following must be true? (A) Lee is assigned to a single (B) Pat sharts a double with another student (C) Robin shares a double with another student (D) Two of the second-year students share a double with each other (E) Neither of the third-year students is assigned to a single4. If Robin is assigred to a triple, which one of the following must be true? (A) Lee is assigned to a single (B) Two second-year students share a double with each other (C) None of the rooms assigned to the students is a single (D) Two of the rooms assigned to the students are singles. (E) Three of the rooms assigned to the students are singles5. If T erry and V al assigned to different doubles from each other, other, then it must be true of the students' rooms that exactly (A) one is a single (B) two are singles (C) two are doubles (D) one is a triple (E) two are triples6. Which one of the following could be true? (A) The two fourth-year students are assigned to singles. (B) The two fourth-year students share a double with cach other. (C) Lee shares a room with a second-year student (D) Lee shares a room with a third-year student (E) Pat shares a triple with two other studentsQuestions 7-11 A worker will insert colored light bulbs into a billboard equipped with exactly three light sockets, which are labled lights 1, 2, and 3. The worker has three green bulbs, three purple bulbs, and three yellow bulbs. Seiection of bulbs for the sockets is governed by the following conditions: Whenever light 1 is purple, light 2 must be yellow. Whenever light 2 is purple, light 1 must be green. Whenever light 3 is either purple or yellow, light 2 must be purple.7. Which one of the following could be an accurate list of the colors of light bulbs selected for lights 1, 2 and 3, respectively? (A) green, green, yellow (B) purple, green, green (C) purple, purple, green (D) yellow, purple, green (E) yellow, yellow, yellow8. If light 1 is yellow, then any of the following can be true, EXCEPT: (A) Light 2 is green. (B) Light 2 is purple (C) Light 3 is green (D) Light 3 is purple (E) Light 3 is yellow9. There is exactly one possible color sequence of the three lights if which one of the following is true? (A) Light 1 is purple. (B) Light 2 is purple. (C) Light 2 is yellow (D) Light 3 is purple. (E) Light 3 is yellow (A) one (B) two (C) three (D) four (E) five11. If no two lights are assigned light bulbs that are the same color as each other, then which one of the following could be true? (A) Light I is green, and light 2 is purple. (B) Light I is green, and light 2 is yellow. (C) Light I is purple, and light 2 is yellow. (D) Light I is yellow, and light 2 is green. (E) Light I is yellow, and light 2 is purple.Questions 12-17 An attorney is scheduling interviews with witnesses for a given week. Monday through Saturday. Two full consecutive days of the week must be reserved for interviewing hostile withesses. In addition, nonhostile witnesses Q, R, U, X, Y, and Z will each be interviewed exactly once for a full morning or afternoon. The only witnesses who will be interviewed simultaneously with each other are Q and R. The following conditions apply. X must be interviewed on Thursday morning Q must be interviewed at some time before X. U must be interviewed at some time before R Z must be interviewed at some time after X and at some time after Y.12. Which one of the following is a sequence, from first to last, in which the nonhostile witnesses could be interviewed? (A) Q with R, U, X, Y, Z (B) Q, U, R, X, with Y, Z (C) U, X, Q, with R, Y, Z (D) U, Y, Q, with R, X, Z (E) X, Q, with U, Z, R, Y13, Which one of the following is acceptable as a complete schedule of witnesses for Tuesday morning. Tuesday afternoon, and Wednesday morning,respectively? (A) Q, R, none (B) R, none, Y (C) U, Y, none (D) U, Y, none (E) Y, Z, none14.If Y is interviewed at some time after X, which one of the following must be a day reserved for interviewing hostile witnesses? (A) Monday (B) Tuesday (C) Wednesday (D) Friday (E) Saturday15. If R is interviewed at some time after Y which one of the following must be a day reserved for interviewing hostile witnesses? (A) Monday (B) Tuesday (C) Wednesday (D) Thursday (E) Friday16. If on Wednesday afternoon and on Monday the attomey conducts no interviews, which one of the following be true? (A) Q is interviewed on the same day as U (B) R is interviewed on the same day as Y (C) Y is interviewed on the same day as U (D) Y is interviewed on the same day as Wednesday (E) Z is interviewed on the same day as Friday17. If Z is interviewed on Saturday morning which one of the following can be true? (A) Wednesday is a day reserved for interiewing hostile witnesses. (B) Friday is a day reserved for interviewing hostile witnesses. (C) R is interviewed on Thursday (D) U is interviewed on Tuesday (E) Y is interviewed at some time before ThursdayQuestions 18-24advertised each week. Exactly one of the products will be a member of two of these four pairs. The following constraints must be observed: J is not advertised during a given week unless H is advertised during the immediately precceding week. The product that is advertised during two of the weeks is advertised during week 4 but is not advertised during week 3 G is not advertised during a given week unless either J or else O is also advertised that week. K is advertised during one of the first two weeks O is one of the products advertised during week 318. Which one of the following could be the schedule of advertisernents? (A) week 1: G, J; week 2: K, L; week 3: O, M; week 4: H, L (B) week 1: H, K; week 2: J, G; week 3: O, L; week 4: M, K (C) week 1: H, K; week 2: J, M; week 3: O, L; week 4: G, M (D) week 1: H, L; week 2: J, M; week 3: O, G; week 4: K, L (E) week 1: K, M; week 2: H, J; week 3: O, G; week 4: L, M19. Which one of the following is a pair of products that CANNOT be advertised during the same week as each other? (A) H and k (B) H and M (C) J and O (D) K and L (E) L and M20. Which one of the following must be advertised during week 2? (A) G (B) J (C) K (D) L (E) M21. Which one of the following CANNOT be the product that is advertised during two of the weeks? (A) G (B) H (C) K (D) L (E) M22. If L is the product that is advertised during two of the weeks, which one of the following is a product that must be advertised during one of the weeks in which L is advertised (A) G (B) H (C) J (D) K (E) M23. Which one of the following is a product that could be advertised in any of the four weeks? (A) H (B) J (C) K (D) L (E) O24. Which one of the following is a pair of products that could be advertised during the same week as each other (A) G and H (B) H and J (C) H and O (D) K and O (E) K and O(F) M and OCBBCA CDAAB ADDBE EABCB AEDEsection 4Time—35 minutes23 QuestionsDirections: Each group of questions in this section is based on a set of conditions. In answering some of the questions it may be useful to draw a rough diagram. Choose the response that most accurately and completely answers each questions and blacken the corresponding space on your answer sheet. In certain recipe contest each contestant submits submits two recipes. one for an appetizer and one for a main dish T ogether the two recipes must include exactly seven flavorings—fenugreek, ginger, lemongrass, nutmeg, paprika, saffron, and turmeric—with no flavoring included in more than one of the two recipes. Each contestant's recipes must satisfy the following conditions The appetizer recipe includes at most three of the flavorings. Fenugreek is not included in the same recipe as nutmeg. Saffron is not included in the same recipe as turmeric Ginger is included in the same recipe as nutmeg1. Which one of the following could be a complete and accurate list of the flavorings included in one contestant's main-dish recipe?(A) fenugreek, lemongrass, saffron(B) fenugreek, ginger, nutmeg, turmeric(C) ginger, lemongrass, nutmeg, paprika(D) ginger, nutmeg, paprika, turmeric(E) lemongass nutmeg, saffron, turmeric2. If a contestant's appetizer recipe does not include fenugreek, then the contestant's appetizer recipe must include(A) ginger(B) lemongrass(C) paprika(D) saffron(E) turmeric3. Which one of the following could be a list of all of the flavorings included in one contestant's appetizer recipe?(A) fenugreek, saffron(B) ginger, nutmeg(C) fenugreek, nutmeg, turmeric(D) lemongrass, nutmeg, saffron(E) fenugreek, lemongrass, paprika, turmeric4. If a contestant includes lemongrass in the same recipe as paprika, which one of the following is a flavoring that must be included in the contestant's main-dish recipe?(A) ginger(B) lemongrass(C) nutmeg(D) saffron(E) turmeric5. If the condition that requires ginger to be included in the same recipe as nutmeg is suspended but all of the other original conditions remain in effect. then which one of the following could be a list of all of the flavorings included in one contestant's main-dish recipe?(A) ginger, lemongrass, nutmeg, paprika(B) ginger, lemongrass, paprika, turmeric(C) fenugreek, ginger, lemongrass, paprika, saffron(D) fenugreek, ginger, lemongrass, saffron, turmeric(D) fenugreek,lemongrass, nutmeg, paprika, saffronQuestions 6-10 Seven singer—Jamine, Ken, Lalitha, Maya, Norton, Olive, and Patrick—will be scheduled to perform in the finals of a singing competition. During the evening of the competition, each singer, performing alone, will give exactly one performance. The schedule for the evening must conform to the following requirements. Jamle performs immediately after Ken Parick performs at some time after Maya Lalitha performs third only if Norton performs fifth. If Patrick does not perform second, he performs fifth6. Which one of the following is an acceptable schedule for the evening's performers, from first through seventh?(A) Ken, Jamie, Maya, Lalitha, Patrick Norton, Olive(B) Lalitha, Patrick, Norton, Olive Maya, Ken, Jamie(C) Norton, Olive Ken, Jamie, Maya, Patrick, Lalitha(D) Olive, Maya, Ken, Lalitha, Patrick, Norton, Jamie(E) Olive, Maya, Lalitha, Norton,Patrick, Ken Jamie7. If Lalitha is scheduled for the third performance which one of the following must be scheduled for the sixth performance?(A) Jame(B) Ken。
美国法学院入学考试LSAT逻辑推理真题精选及详解【圣才出品】
LSAT逻辑推理真题精选及详解Directions: The questions in this section are based on the reasoning contained in brief statements or passages. For some questions, more than one of the choices could conceivably answer the question. However, you are to choose the best answer, that is, the response that most accurately and completely answers the question. You should not make assumptions that are by commonsense standards implausible, superfluous, or incompatible with the passage. After you have chosen the best answer, blacken the corresponding space on your answer sheet.1. Economist: Every business strives to increase its productivity, for this increases profits for the owners and the likelihood that the business will survive. But not all efforts to increase productivity are beneficial to the business as a whole. Often, attempts to increase productivity decrease the number of employees, which clearly harms the dismissed employees as well as the sense of security of the retained employees.Which one of the following most accurately expresses the main conclusion of the economist’s argument?(A) If an action taken to secure the survival of a business fails to enhance the welfare of the business’s employees, that acti on cannot be good for the business as a whole.(B) Some measures taken by a business to increase productivity fail to be beneficial to the business as a whole.(C) Only if the employees of a business are also its owners will the interests of the employees and owners coincide, enabling measures that will be beneficial to the business as a whole.(D) There is no business that does not make efforts to increase its productivity.(E) Decreasing the number of employees in a business undermines the sense of security of retained employees.答案:B解析:选项B译为:商家为提高生产力而采取的有些措施并没有带来企业整体利益。
欧叔智力测试题目(3篇)
第1篇欢迎来到欧叔的智力挑战之旅!在这里,你将面临一系列精心设计的智力测试题目,从逻辑推理到数学难题,从语言游戏到谜题,每一道题目都旨在锻炼你的大脑,提升你的思维能力。
准备好开始这场智慧之旅了吗?请跟随欧叔的脚步,一起挑战自我!第一站:逻辑推理题目1:真假陈述小明、小红和小李三个人的陈述如下:- 小明说:“小红撒谎了。
”- 小红说:“小李在说谎。
”- 小李说:“我不是在说谎。
”请问,谁在说谎?为什么?题目2:数字推理观察以下数列:2, 4, 8, 16, 32, ...请继续写出接下来的四个数字。
题目3:时间谜题如果时钟的时针和分针在3点整时重合,那么它们下一次重合是在几点几分?第二站:数学难题题目4:几何问题一个正方体的边长为5cm,求这个正方体的表面积和体积。
题目5:概率问题一个袋子里有5个红球和3个蓝球,随机取出两个球,求取出两个红球的概率。
题目6:代数问题解方程:2x + 3y = 12,其中x和y都是正整数。
第三站:语言游戏题目7:字母谜题将以下字母序列重新排列成一个有意义的单词:E, L, E, V, E, N, T。
题目8:成语接龙请用成语接龙的方式,从“一诺千金”开始,连续接上五个成语。
题目9:谜语猜猜一个东西,白天用得着,晚上用不着。
是什么?第四站:谜题挑战题目10:智力拼图请将以下数字拼成一个有意义的词语:3, 6, 8, 9, 0。
题目11:数字谜题一个数,去掉它的最后一位,剩下的是它的两倍。
这个数是多少?题目12:逻辑谜题一个房间里有五个人,分别是张三、李四、王五、赵六和孙七。
他们分别来自不同的城市:北京、上海、广州、深圳和杭州。
已知:- 张三不来自北京。
- 李四来自上海。
- 王五来自广州。
- 赵六来自深圳。
- 孙七来自杭州。
请问,谁来自北京?第五站:综合挑战题目13:智力问答以下哪个是著名的“不可能的任务”?A. 穿越时空B. 翻转地球C. 一个人同时成为两个国家元首D. 一个人同时讲两种不同的语言题目14:历史知识以下哪位历史人物是著名的科学家,同时也是一位政治家?A. 牛顿B. 达尔文C. 爱因斯坦D. 马克思题目15:文化常识以下哪个成语与中国的传统节日“端午节”有关?A. 狐假虎威B. 狐狸吃葡萄C. 狐狸请客D. 狐狸吃月饼结束语恭喜你完成了欧叔的智力挑战之旅!在这场旅程中,你不仅锻炼了逻辑思维、数学能力,还提升了语言理解和谜题解决技巧。
- 1、下载文档前请自行甄别文档内容的完整性,平台不提供额外的编辑、内容补充、找答案等附加服务。
- 2、"仅部分预览"的文档,不可在线预览部分如存在完整性等问题,可反馈申请退款(可完整预览的文档不适用该条件!)。
- 3、如文档侵犯您的权益,请联系客服反馈,我们会尽快为您处理(人工客服工作时间:9:00-18:30)。
最新 LSAT 考试逻辑推理模拟试题(三)TEST 2 SECTION II Time 35 minutes 24 Questions Directions: The questions in this section are based on the reasoning contained in brief statements or passages.1.Some people believe that witnessing violence in movies will discharge aggressive energy.Does watching someone else eat fill one’s own stomach? In which one of the following does the reasoning most closely parallel that employed in the passage? (A) Some people think appropriating supplies at work for their own personal use is morally wrong. Isn’t shoplifting morally wrong? (B) Some people think nationalism is defensible. Hasn’t nationalism been the excuse for committing abominable crimes? (C) Some people think that boxing is fixed just because wrestling usually is. Are the two sports managed by the same sort of people? (D) Some people think that economists can control inflation. Can meteorologists make the sun shine?(D) (E) Some people think workaholics are compensating for a lack of interpersonal skills. However, aren’t most doctors workaholics?2.Ann: All the campers at Camp Winnehatchee go to Tri-Cities High SchoolBill: That’s not true. Some Tri-Cities students are campers at Camp Lakemont. Bill’s answer can be best explained on the assumption that he has interpreted Ann’s remark to mean that (A) most of the campers at Camp Lakemont come from high schools other than Tri-Cities (B) most Tri-Cities High School students are campers at Camp Winnehatchee (C) some Tri-Cities High School students have withdrawn from Camp Lakemont (D) all Tri-Cities High School students have withdrawn from Camp Lakemont(E) (E) only campers at Camp Winnehatchee are students at Tri-Cities High School3.More than a year ago, the city announced that police would crack down on illegally parkedcars and that resources would be diverted from writing speeding tickets to ticketing illegally parked cars. But no crackdown has taken place. The police chief claims that resources have had tobe diverted from writing speeding tickets to combating the city’s staggering drug problem. Yet the police are still writing as many speeding tickets as ever. Therefore, the excuse about resources being tied up in fighting drug-related crime simply is not true. The conclusion in the passage depends on the assumption that (A) every member of the police force is qualified to work on combating the city’s drug problem (B) drug-related crime is not as serious a problem for the city as the police chief claims it is (C) writing speeding tickets should be as important a priority for the city as combating drug-related crime (D) the police could be cracking down on illegally parked cars and combating the drug problem without having to reduce writing speeding tickets(E) (E) the police cannot continue writing as many speeding tickets as ever while diverting resources to combating drug-related crime4.Dried grass clippings mixed into garden soil gradually decompose, providing nutrients forbeneficial soil bacteria. This results in better-than-average plant growth. Yet mixing fresh grass clippings into garden soil usually causes poorer-than-average plant growth. Which one of the following, if true, most helps to explain the difference in plant growth described above? (A) The number of beneficial soil bacteria increases whenever any kind of plant material is mixed into garden soil. (B) Nutrients released by dried grass clippings are immediately available to beneficial soil bacteria. (C) Some dried grass clippings retain nutrients originally derived from commercial lawn fertilizers, and thus provide additional enrichment to the soil. (D) Fresh grass clippings mixed into soil decompose rapidly, generating high levels of heat that kill beneficial soil bacteria.(D) (E) When a mix of fresh and dried grass clippings is mixed into garden soil, plant growth often decreases.5.A gas tax of one cent per gallon would raise one billion dollars per year at currentconsumption rates. Since a tax of fifty cents per gallon would therefore raise fifty billion dollars per year, it seems a perfect way to deal with the federal budget deficit. This tax would have the additional advantage that the resulting drop in the demand for gasoline would be ecologically sound and would keep our country from being too dependent on foreign oil producers. Which one of the following most clearly identifies an error in the author’s reasoning? (A) The author cites irrelevant data. (B) The author relies on incorrect current consumption figures. (C) The author makes incompatible assumptions.(D) The author mistakes an effect for a cause.(C) (E) The author appeals to conscience rather than reason.6.As symbols of the freedom of the wilderness, bald eagles have the unique capacity to inspirepeople and foster in them a sympathetic attitude toward the needs of other threatened species. Clearly, without that sympathy and the political will it engenders, the needs of more obscure species will go unmet. The conservation needs of many obscure species can only be met by beginning with the conservation of this symbolic species, the bald eagle. Which one of the following is the main point of the passage as a whole? (A) Because bald eagles symbolize freedom, conservation efforts should be concentrated on them rather than on other, more obscure species. (B) The conservation of bald eagles is the first necessary step in conserving other endangered species. (C) Without increased public sympathy for conservation, the needs of many symbolic species will go unmet. (D) People’s love of the wilderness can be used to engender political support for conservation efforts.(B) (E) Other threatened species do not inspire people or foster sympathy as much as do bald eagles. 来源:考试大-LSAT 7. There is no reason why the work of scientists has to be officially confirmed before beingpublished. There is a system in place for the confirmation or disconfirmation of scientific finding, namely, the replication of results by other scientists. Poor scientific work on the part of any one scientist, which can include anything from careless reporting practices to fraud, is not harmful. It will be exposed and rendered harmless when other scientists conduct the experiments and obtain disconfirmatory results. Which one of the following, if true, would weaken the argument? (A) Scientific experiments can go unchallenged for many years before they are replicated. (B) Most scientists work in universities, where their work is submitted to peer review before publication. (C) Most scientists are under pressure to make their work accessible to the scrutiny of replication. (D) In scientific experiments, careless reporting is more common than fraud.(A) (E) Most scientists work as part of a team rather than alone.8.Alice: Quotas on automobile imports to the United States should be eliminated. Thendomestic producers would have to compete directly with Japanese manufacturers and would be forced to produce higher-quality cars. Such competition would be good for consumers. David: You fail to realize, Alice, that quotas on automobile imports are pervasive worldwide.Since German, Britain, and France have quotas, so should the United States. Which one of the following most accurately characterizes David’s response to Alice’s statement? (A) David falsely accuses Alice of contradicting herself. (B) David unfairly directs his argument against Alice personally. (C) David uncovers a hidden assumption underlying Alice’s position. (D) David takes a position that is similar to the one Alice has taken.(E) (E) David fails to address the reasons Alice cites in favor of her conclusion.9.Governments have only one response to public criticism of socially necessary services:regulation of the activity of providing those services. But governments inevitably make the activity more expensive by regulating it, and that is particularly troublesome in these times of strained financial resources. However, since public criticism of child-care services has undermined all confidence in such services, and since such services are socially necessary, the government is certain to respond. Which one of the following statements can be inferred from the passage? (A) The quality of child care will improve. (B) The cost of providing child-care services will increase. (C) The government will use funding to foster advances in child care. (D) If public criticism of policy is strongly voiced, the government is certain to respond.(B) (E) If child-care services are not regulated, the cost of providing child care will not increase. Advertisers are often criticized for their unscrupulous manipulation of people’s tastes and10.wants. There is evidence, however, that some advertisers are motivated by moral as well as financial considerations. A particular publication decided to change its image from being a family newspaper to concentrating on sex and violence, thus appealing to a different readership. Some advertisers withdrew their advertisements from the publication, and this must have been because they morally disapproved of publishing salacious material. Which one of the following, if true, would most strengthen the argument? (A) The advertisers switched their advertisements to other family newspapers. (B) Some advertisers switched from family newspapers to advertise in the changed publication. (C) The advertisers expected their product sales to increase if they stayed with the changed publication, but to decrease if they withdrew. (D) People who generally read family newspapers are not likely to buy newspapers that concentrate on sex and violence.(C) (E) It was expected that the changed publication would appeal principally to those in a different income group.11. “If the forest continues to disappear at its present pace, the koala will approach extinction,” said the biologist. “So all that is needed to save the koala is to stop deforestation,” said the politician. Which one of the following statements is consistent with the biologist’s claim but not with the politician’s claim? (A) Deforestation continues and the koala becomes extinct. (B) Deforestation is stopped and the koala becomes extinct. (C) Reforestation begins and the koala survives. (D) Deforestation is slowed and the koala survives.(B) (E) Deforestation is slowed and the koala approaches extinction.12. People have long been fascinated by the paranormal. Over the years, numerous researchers have investigated telepathy only to find that conclusive evidence for its existence has persistently evaded them. Despite this, there are still those who believe that there must be “something in it” since some research seems to support the view that telepathy exist. However, it can often be shown that other explanations that do comply with known laws can be given. Therefore, it is premature to conclude that telepathy is an alternative means of communication. In the passage, the author (A) supports the conclusion by pointing to the inadequacy of evidence for the opposite view (B) supports the conclusion by describing particular experiments (C) supports the conclusion by overgeneralizing from a specific piece of evidence (D) draws a conclusion that is not supported by the premises(A) (E) rephrases the conclusion without offering any support for it 来源:考试大-LSAT 13. If retail stores experience a decrease in revenues during this holiday season, then either attitudes toward extravagant gift-giving have changed or prices have risen beyond the level most people can afford. If attitudes have changed, then we all have something to celebrate this season. If prices have risen beyond the level most people can afford, then it must be that salaries have not kept pace with rising prices during the past year. Assume the premises above to be true. If salaries have kept pace with rising prices during the past year, which one of the following must be true? (A) Attitudes toward extravagant gift-giving have changed. (B) Retail stores will not experience a decrease in retail sales during this holiday season. (C) Prices in retail stores have not risen beyond the level that most people can afford during this holiday season. (D) Attitudes toward extravagant gift-giving have not changed, and stores will not experience a decrease in revenues during this holiday season.(C)(E) Either attitudes toward extravagant gift-giving have changed or prices have risen beyond the level that most people can afford during this holiday season. 14. The “suicide wave” that followed the United States stock market crash of October 1929 is more legend than fact. Careful examination of the monthly figures on the causes of death in 1929 shows that the number of suicides in October and in November was comparatively low. In only three other months were the monthly figures lower. During the summer months, when the stock market was flourishing, the number of suicides was substantially higher. Which one of the following, if true, would best challenge the conclusion of the passage? (A) The suicide rate is influenced by many psychological, interpersonal, and societal factors during any given historical period. (B) October and November have almost always had relatively high suicide rates, even during the 1920s and 1930s. (C) The suicide rate in October and November of 1929 was considerably higher than the average for those months during several preceding and following years. (D) During the years surrounding the stock market crash, suicide rates were typically lower at the beginning of any calendar year than toward the end of that year.(C) (E) Because of seasonal differences, the number of suicides in October and November of 1929 would not be expected to be the same as those for other months.15. A well-known sports figure found that combining publicity tours with playing tours led to problems, so she stopped combining the two. She no longer allows bookstore appearances and playing in competition to occur in the same city within the same trip. This week she is traveling to London to play in a major competition, so during her stay in London she will not be making any publicity appearances at any bookstore in London. Which one of the following most closely parallels the reasoning used in the passage? (A) Wherever there is an Acme Bugkiller, many wasps are killed. The Z family garden has an Acme Bugkiller, so any wasps remaining in the garden will soon be killed. (B) The only times that the hospital’s emergency room staff attends to relatively less serious emergencies are times when there is no critical emergency to attend to. On Monday night the emergency room staff attended to a series of fairly minor emergencies, so there must not have been any critical emergencies to take care of at the time. (C) Tomato plants require hot summers to thrive. Farms in the cool summers of country Y probably do not have thriving tomato plants. (D) Higher grades lead to better job opportunities, and studying leads to higher grades. Therefore, studying will lead to better job opportunities.(B) (E) Butter knives are not sharp. Q was not murdered with a sharp blade, so suspect X’s butterknife may have been the murder weapon.Questions 16-17 The advanced technology of ski boots and bindings has brought a dramatic drop in the incidence of injuries that occur on the slopes of ski resorts: from 9 injuries per 1,000 skiers in 1950 to 3 in 1980. As a result, the remainder of ski-related injuries, which includes all injuries occurring on the premises of a ski resort but not on the slopes, rose from 10 percent of all ski-related injuries in 1950 to 25 percent in 1980. The incidence of these injuries, including accidents such as falling down steps, increases with the amount of alcohol consumed per skier.16. Which one of the following can be properly inferred from the passage? (A) As the number of ski injuries that occur on the slopes decreases, the number of injuries that occur on the premises of ski resorts increases. (B) The amount of alcohol consumed per skier increased between 1950 and 1980. (C) The technology of ski boots and bindings affects the incidence of each type of ski-related injury. (D) If the technology of ski boots and bindings continues to advance, the incidence of ski-related injuries will continue to decline.(E) (E) Injuries that occurred on the slopes of ski resorts made up a smaller percentage of ski-related injuries in 1980 than in 1950.17. Which one of the following conflicts with information in the passage? (A) The number of ski injuries that occurred on the slopes was greater in 1980 than in 1950. (B) A skier was less likely to be injured on the slopes in 1950 than in 1980. (C) The reporting of ski injuries became more accurate between 1950 and 1980. (D) The total number of skiers dropped between 1950 and 1980.(B) (E) Some ski-related injuries occurred in 1980 to people who were not skiing.18. Learning how to build a nest plays an important part in the breeding success of birds. For example, Dr. Snow has recorded the success of a number of blackbirds in several successive years. He finds that birds nesting for the first time are less successful in breeding than are older birds, and also less successful than they themselves are a year later. This cannot be a mere matter of size and strength, since blackbirds, like the great majority of birds, are fully grown when they leave the nest. It is difficult to avoid the conclusion that they benefit by their nesting experience. Which one of the following, if true, would most weaken the argument? (A) Blackbirds build better nests than other birds. (B) The capacity of blackbirds to lay viable eggs increases with each successive trial during thefirst few years of reproduction. (C) The breeding success of birds nesting for the second time is greater than that of birds nesting for the first time. (D) Smaller and weaker blackbirds breed just as successfully as bigger and stronger blackbirds. (B) (E) Up to 25 percent of all birds are killed by predators before they start to nest. 来源:考试大-LSAT 19. How do the airlines expect to prevent commercial plane crashes?Studies have shown that pilot error contributes to two-thirds of all such crashes. To address this problem, the airlines have upgraded their training programs by increasing the hours of classroom instruction and emphasizing communication skills in the cockpit. But it is unrealistic to expect such measures to compensate for pilots’ lack of actual flying time. Therefore, the airlines should rethink their training approach to reducing commercial crashes. Which one of the following is an assumption upon which the argument depends? (A) Training programs can eliminate pilot errors. (B) Commercial pilots routinely undergo additional training throughout their careers. (C) The number of airline crashes will decrease if pilot training programs focus on increasing actual flying time. (D) Lack of actual flying time is an important contributor to pilot error in commercial plane crashes.(D) (E) Communication skills are not important to pilot training programs.20. All savings accounts are interest-bearing accounts. The interest from some interest-bearing accounts is tax-free, so there must be some savings accounts that have tax-free interest. Which one of the following arguments is flawed in a way most similar to the way in which the passage is flawed? (A) All artists are intellectuals. Some great photographers are artists. Therefore, some great photographers must be intellectuals. (B) All great photographers are artists. All artists are intellectuals. Therefore, some great photographers must be intellectuals. (C) All great photographers are artists. Some artists are intellectuals. Therefore, some great photographers are intellectuals. (D) All great photographers are artists. Some great photographers are intellectuals. Therefore, some artists must be intellectuals.(C) (E) All great photographers are artists. No artists are intellectuals. Therefore, some great photographers must not be intellectuals.21. One method of dating the emergence of species is to compare the genetic material of relatedspecies. Scientists theorize that the more genetically similar two species are to each other, the more recently they diverged from a common ancestor. After comparing genetic material from giant pandas, red pandas, raccoons, coatis, and all seven bear species, scientists concluded that bears and raccoons diverged 30 to 50 million years ago. They further concluded that red pandas separated from the ancestor of today’s raccoons and coatis a few million years later, some 10 million years before giant pandas diverged from the other bears. Which one of the following can be properly inferred from the passage? (A) Giant pandas and red pandas are more closely related than scientists originally thought they were. (B) Scientists now count the giant panda as the eighth species of bear. (C) It is possible to determine, within a margin of just a few years, the timing of divergence of various species. (D) Scientists have found that giant pandas are more similar genetically to bears than to raccoons. (D) (E) There is substantial consensus among scientists that giant pandas and red pandas are equally related to raccoons.Questions 22-23 Despite improvements in treatment for asthma, the death rate form this disease has doubled during the past decade from its previous rate. Two possible explanations for this increase have been offered. First, the recording of deaths due to asthma has become more widespread and accurate in the past decade than it had been previously. Second, there has been an increase in urban pollution. However, since the rate of deaths due to asthma has increased dramatically even in cities with long-standing, comprehensive medical records and with little or no urban pollution, one must instead conclude that the cause of increased deaths is the use of bronchial inhalers by asthma sufferers to relieve their symptoms.22. Each of the following, if true, provides support to the argument EXCEPT: (A) Urban populations have doubled in the past decade. (B) Records of asthma deaths are as accurate for the past twenty years as for the past ten years. (C) Evidence suggests that bronchial inhalers make the lungs more sensitive to irritation by airborne pollen. (D) By temporarily relieving the symptoms of asthma, inhalers encourage sufferers to avoid more beneficial measures.(A) (E) Ten years ago bronchial inhalers were not available as an asthma treatment.23. Which one of the following is an assumption on which the argument depends?(A) Urban pollution has not doubled in the past decade. (B) Doctors and patients generally ignore the role of allergies in asthma. (C) Bronchial inhalers are unsafe, even when used according to the recommended instructions. (D) The use of bronchial inhalers aggravates other diseases that frequently occur among asthma sufferers and that often lead to fatal outcomes even when the asthma itself does not.(E) (E) Increased urban pollution, improved recording of asthma deaths, and the use of bronchial inhalers are the only possible explanations of the increased death rate due to asthma.24. There is little point in looking to artists for insights into political issues. Most of them hold political views that are less insightful than those of any reasonably well-educated person who is not an artist. Indeed, when taken as a whole, the statements made by artists, including those considered to be great, indicate that artistic talent and political insight are rarely found together. Which one of the following can be inferred from the passage? (A) There are no artists who have insights into political issues. (B) A thorough education in art makes a person reasonably well educated. (C) Every reasonably well-educated person who s not an artist has more insight into political issues than any artist. (D) Politicians rarely have any artistic talent.(E) (E) Some artists are no less politically insightful than some reasonably well-educated persons who are not artists. 来源:考试大-LSAT。